Плитка и... Из дерева Выбор покрытия

Екатерина Шульман: «Если вы разобщены, вас не существует. Гибка, как гусеница, гибридная россия Екатерина шульман последние интервью

Доцент Института общественных наук РАНХиГС Екатерина Шульман изучает гибридные режимы: внешне - демократические, внутренне - нет. Другие известные исследователи этой темы (например, Владимир Гельман или Сергей Гуриев) теперь работают за границей. Это хорошо иллюстрирует, как изменилось политическое устройство России: коммунистический режим своих исследователей за границу не выпускал. При этом граждане РФ свой государственный механизм представляют плохо - это как раз не изменилось. Данное интервью, первоначально размещенное на «Росбалте» вызвало огромный резонанс в России и оно, несомненно, представляет интерес для украинской аудитории.

Знаете, Екатерина Михайловна, поскольку термин «гибридный режим» - новый, неустоявшийся… Употребляют и «частичная демократия», и «пустая демократия», и «иллиберальная демократия»… Предлагаю простую вещь. Я буду перечислять страны, а вы будете говорить - это гибридный режим или нет. Итак: Сингапур, Китай, Россия, Южная Корея…

Тогда уточнение. Любая научная классификация условна. Разложить страны по корзинкам - означает упростить ситуацию. Но без классификации наука жить не может. Научный консенсус сегодня в том, что входным билетом в волшебный клуб стран-гибридов являются многопартийность и регулярные выборы. Как бы ни был авторитарен режим, если есть хотя бы две партии, и они могут принимать участие в выборах, которые проходят в определенные законом сроки, - страна уже не считается классической автократией, диктатурой или тиранией.

Поэтому Китай, где всего одна партия, не является гибридом или «конкурентным авторитаризмом» - это еще один термин, придуманный Стивеном Левицки и Люканом Вэем, написавшими книгу Competitive Authoritarianism: Hybrid Regimes after the Cold War. Кстати, ее обложка украшена изображением российского милиционера, который бьет демонстранта…

Образцовыми гибридами считаются Россия и Венесуэла.

А вот Сингапур - это не гибрид, а гораздо более откровенная автократическая система с фактической однопартийностью. И Южная Корея не подходит, потому что там есть и выборы, и многопартийность, и конкурентные СМИ носят не имитационный, а институциональный характер.

Но я еще раз подчеркну: мы не можем, как биологи, строго проводить границы видов. При этом мы должны заниматься классификацией, выявляя различия и сходства между политическими режимами. А теперь давайте продолжим ваш список…

- Казахстан, Киргизия?..

Да, да! Это гибриды. Есть различные партии, некоторая выборность… Правда, последние события в Казахстане и попытки перейти к «вечному правлению» ставят страну на грань с автократией. Но пока еще - гибриды.

- Беларусь?

Нет. Там нет регулярных выборов и многопартийность практически ликвидирована.

- Турция?

Да, гибрид, без сомнения.

- Таджикистан и Туркмения?

Нет, чистые автократии.

- Иран и Ирак?

Ирак - это failed state, распавшееся государство. А Иран иногда называют теократической демократией - это не гибрид, там не изображают западные демократические институты, там нет выборной ротации. Но если власть в Иране будет переходить от разнообразных стражей революции и религиозных деятелей к выборным органам, это будет движение в сторону гибридности.

- И, наконец, Украина.

Украина - это так называемая анократия, или слабое государство. Украина не похожа на Россию вообще ничем, она выбивается из постсоветской государственной матрицы. Но слабое государство - это перекресток больших возможностей. Украину может понести как в сторону failed state, так и в сторону демократии. Пока это политическая система со слабо выраженным государственным центром, у гибридов сила государственных аппаратов обычно выше.

Когда-то, заинтересовавшись фашистскими режимами ХХ века, я обратил внимание, что они возникали только там, где перед этим рухнула монархия. Можно предположить, что фашизм - это такая болезнь перехода от монархии к демократии, строй, когда лидер пытается, по образцу монархии, закрепиться у власти навечно. А поскольку он не монарх, то использует другой инструментарий - например, поощряет худшие инстинкты людей. Это не очень годится как научное определение, но, думаю, понятно. Можно ли аналогичным образом описать появление гибридных режимов, которые, как и фашизм в 1930-х годах, сегодня буквально всюду?

Макс Вебер выделял три основания, по которым власть признается легитимной со стороны управляемых масс: традиционное монархическое основание, харизматическое революционное и процедурное. Монархический тип легитимации основан на традиции и признании священной воли Божьей. Харизматическая легитимация характерна для революционных лидеров: я правлю, потому что я великий вождь и учитель, меня волна революции вынесла! Нетрудно заметить, что харизматический тип является плодом распада религиозного сознания. То есть в Бога мы уже не верим, но еще готовы верить в сверхчеловека. В Гитлера, в Ленина, в Муссолини: «Сей муж судьбы, сей странник бранный, пред кем унизились цари». Это действительно переходная модель на пути распада религиозного сознания как массового явления. В этом смысле появление гибридных режимов является плодом следующего перехода…

- К процедурному типу легитимации.

Да. Процедурный тип называется правовым - это красивый термин. Или бюрократическим - это менее красивый термин. «Я правлю, потому что я прошел определенную процедуру». Грубо говоря, собрал документы, произвел описанные в законе манипуляции - вот поэтому я руководитель на тот срок, который в законе прописан. Поскольку в Бога и героев мы больше не верим, то начинаем верить в закон и процедуры. И сейчас большинство населения Земли живет не при демократиях и не при тоталитарных моделях, которые почти сошли с исторической сцены, а при гибридном правлении. Просто если раньше фашизоидный лидер изображал монарха, насколько хватало доверчивости у народа, то сейчас гибриды изображают демократии. Потому что это необходимо, чтобы быть легитимным в современном мире.

- Чем принципиально режим Владимира Путина отличается от режима Иосифа Сталина?

Да абсолютно всем! Сходства вообще ни в чем не наблюдается, кроме попыток пропаганды сымитировать нечто такое, что отвечает, как им кажется, ностальгическому запросу общества. Хотя этот самый ностальгический запрос они же обществу и навязывают.

Но экономическая модель - принципиально другая. Структура общества - тоже. Демографическая пирамида выглядит совершенно иначе. Кадровый механизм, построение властных органов, - совсем все другое! Попытки провести параллели всегда кажутся мне очень дурным способом научного анализа. То сходство, которое вы замечаете, в основном поверхностное, а самое главное вы упускаете, поскольку самое главное - не сходства, а различия. Я против проведения исторических параллелей - они уводят в сторону.

Но сейчас пишут многие, от Белковского до Павловского, что Путин все главные кадровые решения стал принимать единолично, отказавшись от коллегиальности. То есть случился откат от Брежнева к Сталину.

Я считаю, что это абсолютно неверное мнение. Оно не основано ни на чем, кроме стремления комментаторов поспекулировать на общественных страхах, а особенно на страхах образованного сословия, которому палец покажи - оно видит Сталина. Эти страхи культивируются и официальной пропагандой: просто то, что широким массам продается в виде сентиментальной сказки, интеллигенции продают в виде пугала. Соответственно, обе аудитории довольны, каждая по-своему.

Что происходит на самом деле? Мы действительно видим осень, ну, или зрелость нашего гибридного режима. По статистике, средний срок жизни персоналистских автократий (а, скажем, исследователь Барбара Геддес классифицирует Россию именно как personalist autocracy) - 15 лет. Потом у них наступает период трансформации, и чаще всего не в сторону единоличного правления.

Наше 15-летие случилось в 2014 году. После этого с режимом действительно стали происходить всякие интересные вещи, за которыми вся мировая политическая наука с большим вниманием следит. Мы видим ухудшение экономической ситуации и сокращение той ренты, раздачей которой режим жил. Основа политического режима такого типа - это покупка лояльности масс и элит. Рента сжимается и, соответственно, системе нужно менять свой способ бытования. А она не хочет меняться.

Но в чем благословение гибридности? Она более гибка и адаптивна, чем автократия. Гибрид, как гусеница, может переползти тот порог, о который разбиваются автократии, - в силу того, что он такой мягкий, неопределенный, кольчатый и может имитировать практически любую форму. И вместо того, чтобы за кадровыми решениями Путина видеть сталинизм, разумнее увидеть за этим попытку системы избавиться от плохих управленцев, на которых нет больше денег. Их нужно заменить теми, кто, как кажется системе, дешевле и эффективнее. И это не воля какого-то конкретного человека. У системы есть свой коллективный разум: она хочет сохраниться. А поскольку это все-таки не демократия, и у нее нет ни нормальной ротации, ни кадровых лифтов, то новых управленцев она берет неподалеку.

Это не Сталин, который, когда занимался кадровыми перестановками, сопровождал это кровавой мясорубкой, да еще подводил под это публично декларируемую идеологическую платформу. А у нас даже кампания по борьбе с коррупцией толком не заявлена, все какие-то единичные случаи… То один губернатор вроде нехороший, то другой. А вот министр обороны вроде тоже был нехороший, но потом для него все кончилось благополучно. Это не диктатор железной рукой проводит свою политику. Это, цитируя Гоббса, «война всех против всех» в самом классическом виде: грызня кланов. И у верховного правителя задача одна - поддерживать баланс столько, сколько хватит сил.

Есть, кстати, большая полемика в западной политической науке - правильно ли мы делаем, что навязываем демократические институты странам третьего мира, не продлеваем ли мы этим жизнь их автократам? Потому что будь они чистыми автократами, которые врагов зашивают в мешок и кидают в Босфор, они бы уже пали жертвой восстания и переворота.

Мое мнение - печалиться не о чем. Может, коллективный Запад и продлевает жизнь автократам, но он делает их куда менее кровожадными и опасными для их собственных народов.

Идеологическая пустота - насколько она характерна для всех гибридов? Или это только в России так сложилось? И может ли у нас идеология появиться в виде «национальной идеи»?

И «так сложилось», и объективная необходимость. Поскольку цель гибридного режима - не завоевание мира, а всего лишь собственное выживание, то он не может позволить себе путы идеологии. Режим должен быть свободен и говорить нечто невнятное, чтобы в любой момент отползти назад или прыгнуть чуть вперед с целью самосохранения.

Посмотрите, например, на Турцию. Какая там идеология? Вроде бы, кемалистская, то есть светское государство. А вроде бы - и немножко исламистская. А была совсем исламистская, когда дружили с Гюленом, а потом с Гюленом разругались, но об исламе продолжили говорить… Поскольку простому народу это нравится, то надо быть толерантнее к религиозным общинам, - не так, как при Ататюрке… Но при этом пользоваться языком демократии, когда надо бороться с военным переворотом и собрать массовый митинг… Вот что такое прекрасная гибкость гибридов!

А то, что у нас постоянно возникают разговоры о необходимости сформулировать национальную идеологию, не означает, что мы к ней приблизились. В 2014 году, максимально турбулентном, у нас произошло максимальное приближение к официально декларируемой идеологической доктрине. Это была доктрина Русского мира и какого-то такого православного имперства. Но как только это стало влиять на действия власти (ведь если мы - православные имперцы, то мы должны присоединить к России Донбасс), данное направление было закрыто. Даже терминология исчезла.

Почему? Именно потому, что если ты исповедуешь некую идеологию, то она запрещает тебе делать ряд вещей (например, есть свинину и работать по пятницам), и она предписывает тебе делать ряд вещей (скажем, молиться пять раз в день). Если ты исповедуешь доктрину толерантности, прав человека, демократии, свободного рынка, то ты тоже обязан делать определенные вещи, а если делаешь противоположные вещи, например, поддерживаешь близкие отношения с теократическим авторитарным режимом, таким, как Саудовская Аравия, то в тебя начинают тыкать палкой и говорить: «Как же так, вы за права женщин и меньшинств, при этом ваши лучшие друзья в регионе - это те, кто побивает камнями за супружескую измену?!» Это тоже идеологическое ограничение! А гибриды стремятся к свободе от ограничений.

- Возможно ли перерождение гибридного режима в чистую автократию?

Если обратиться к Левицки и Вэю, то есть три фактора, которые направляют гибридные режимы на ту или иную дорогу. Первый - это leverage, т. е. влияние, которое оказывает на страну ее ближайший крупный торговый и финансовый партнер. Если этот партнер - демократия, то, соответственно, режим будет демократизироваться, а если диктатура, то он должен либо тоже стать диктатурой, либо развалиться, стать failed state. Второй фактор - это linkage, то есть вовлеченность. Это то, насколько режим изолирован - или, наоборот, втянут в отношения со всем остальным миром. И третий - это внутренняя организационная структура. Это то, насколько режим строит у себя демократические институты, даже если они не совсем работают. Чем больше он их построил - тем больше шансов демократизироваться. И это то, насколько у него эффективный правоохранительный и репрессивный аппараты. Они будут гнуть режим в авторитарном направлении.

На практике довольно мало случаев, когда гибрид преобразуется в тиранию. Лично я такое пока вижу только в Белоруссии. Но в основном не для того гибриды становятся гибридами, чтобы обменять это счастливое существование на тоталитарную крепость. Потому что они для того все имитируют, чтобы ездить в развитые страны, чтобы торговать и получать товары… Они не хотят обратно. Они чувствуют угрозу. А концентрация власти, при всей ее соблазнительности, ведет к тому, что ты становишься легкой жертвой либо переворота, либо массовых волнений. После чего наступает этап failed state, но это уже другая история.

Посмотрите на Турцию, переживающую сильную турбулентность. У нас ждут, что она превратится в диктатуру Эрдогана. А я думаю, что этого не будет. Есть ряд научных работ о том, как перевороты (в том числе неудачные) приводят, как ни парадоксально, к последующей демократизации режима. Потому что тот, кому они угрожают, чтобы удержаться у власти, вынужден опираться на какие-то другие кланы и страты, нежели те, которые взбунтовались против него. То есть он так или иначе вынужден делиться с кем-то властью. Этот конкурентный авторитаризм может пульсировать. Он может подмораживаться, а потом демократизироваться, но сохранять при этом свои базовые свойства: имитационность демократических процедур, отсутствие истинной концентрации власти, экономических ресурсов и отсутствие серьезной машины репрессий.

- Значит ли это, что репрессии - на уровне Брежнева, а не Сталина - более невозможны?

Реалии делают их ненужными. Чтобы запугать общество, достаточно одного показательного процесса, который покажут все телеканалы и про который напишут все СМИ и социальные сети. Кроме того, промежуточные автократии, в отличие от тоталитарных структур прошлого, не стремятся удержать недовольных граждан - они никогда не ограничивают выезд за границу. Они запугивают ту часть общества, которой одновременно говорят: «А, валите! Без вас будет спокойнее!» В Турции то же самое происходит…

- Ну да, нобелевский лауреат Орхан Памук уехал в Америку…

Совершенно верно. Это же достаточно свободная экономика. Значит, деньги можно удаленно зарабатывать. А раз так, то зачем жить в неуютной стране, где происходят неприятные вещи? Вот почему нынешние режимы не устраивают у себя кровавую баню. Слава богу - и это вообще показатель политического прогресса. В мире уровень насилия, как известно, вообще снижается…

Про снижение насилия в мире пишет Стивен Пинкер в The Better Angels of our Nature, у нас на том же настаивает Акоп Назаретян, автор теории техно-гуманитарного баланса…

Да. И все то, чему мы сейчас ужасаемся, те же события в Сирии, в терминах Второй мировой войны - просто день, когда ничего не случилось. Но поскольку благодаря «Ютьюбу» и телевизору мы все это видим крупным планом, то нам оно ужасающе наглядно представляется…

А с точки зрения насилия гораздо большую опасность представляют как раз failed states. Посмотрим на распад Югославии, на ту же Сирию, на агонию в тех африканских странах, где распад начинается… И посмотрим, что сейчас будет в Венесуэле: с точки зрения ученого, это очень интересно. Венесуэла приближается к порогу failed state. Я надеюсь, что окружение из стран более, скажем так, удачливых, чем она сама, и нахождение в зоне интересов Соединенных Штатов спасет ее от большой беды.

- Угроза failed state реальна для России?

Пока нет. Венесуэлу связывала как раз некоторая идеологичность. Там была идея Симона Боливара, идея народного социализма - соответственно, венесуэльцы были вынуждены проводить определенную политику, которая привела их к сегодняшнему результату. Ограничивать конкуренцию, цены, раздавать гражданам деньги и товары, бороться со спекуляцией… Вот эта страшная левизна - в благословенном климате, в невоюющей стране - привела к тому, что Венесуэла ухитрилась устроить у себя голод. Ни климат, ни ресурсы не имеют такого значения, как политические институты. Они способны устроить рай и ад в любой точке Земли.

Так вот: нам failed state не грозит, потому что у нас более многоукладная экономика, корыстная правящая элита, которая не хочет никаких ограничений, - соответственно, я не вижу причин для венесуэльского сценария.

- В чем страновая специфика России?

Если мы посмотрим на списки гибридов, которые все исследователи составляют, - у Левицки и Вэя их 35, у Барбары Геддес 128, у кого-то их вообще под 150, - возникает ощущение, что значение для таких стран имеет только одна вещь. Если они расположены в Латинской Америке или Восточной Европе, то будут демократизироваться. А если в Африке или на постсоветском пространстве - то, скорее всего, стагнировать и разваливаться.

Проблема России в том, что она сама себе является значимым партнером. Она настолько велика, что влияет на пространство вокруг себя - и одновременно подвержена ему же. Как Китай. В политологии недемократическая страна, которая вовлекает в свою орбиту другие страны, толкая их по авторитарному пути, называется «черным рыцарем». Так вот, Россия - сама себе «черный рыцарь». И это одна из ее особенностей. Мы как бы и сами трансформируемся, и на других желаем влиять - и влияем.

У нас есть все предпосылки для демократического развития. Костяк нашей конституционной системы, несмотря на изменения последних лет, достаточно здоровый. У нас есть институты, подобные институтам развитых стран, и не все из них являются декорацией. У нас в основном городское население. У нас отсутствует так называемый демографический навес - большая молодежная страта, которая ассоциируется в демографии с высоким уровнем насилия. У нас основная страта - это возраст 40+. Что говорит в пользу мирной жизни и поступательного развития. Но, правда, это же не дает нам сильно прогрессировать и модернизироваться.

Впрочем, я не уверена, что мы хотим прогрессировать скачками. Надо помнить, что тоталитарные модели часто именно прогрессистские и модернизационные. Они тащат всех силком в светлое будущее. Уникальность нашей ситуации в том, что, кажется, руку протяни - и дотянешься до здорового развития. Что буквально полтора оборота недокрученных остается в системе, чтобы стать если не сияющим градом на холме, то вполне работающей административной машиной. Но этих полутора оборотов каждый раз и не хватает.

Я думаю, что Россия будет эволюционировать как под влиянием общественного запроса, так и под давлением обстоятельств. Те административные телодвижения, которые мы наблюдаем, - это тоже ответ на давление. Система отвечает, как может. И с коррупцией пытается как-то бороться. И заменять управленцев. И по одежке протягивать ножки, потому что денег стало меньше. Система чувствует леденящее дыхание кризиса и реальности.

- Как гибридные режимы решают проблему передачи власти?

Это их роковая проблема, Кощеева игла. Все их сложности концентрируются вокруг этого. Нет у гибридов легального механизма передачи власти! Если бы он был, они были бы демократиями. Они решают эту проблему кто во что горазд. Наиболее находчивые - те, где имеются доминирующие партии. Например, Мексика. Правящая партия выигрывает выборы раз за разом, а внутри выращивает свою элиту и передает постепенно власть новым поколениям функционеров, прошедших, тем не менее, некоторую выборную тренировку. Это немножко вариант КПСС, но без фанатизма. При этом другие партии тоже существуют, и они какие-то доли тоже могут выигрывать. Этот партийный механизм позволяет режиму сохраняться практически бесконечно, и если есть секрет долголетия, то он в этом.

Наиболее ломкие из режимов - персоналистские. Они концентрируют власть в руках одного человека и его ближайшего окружения, а дальше начинаются мучения: наследник, преемник… Дети есть - детей нет… А преемника нельзя предъявлять слишком рано, чтобы его не сожрали другие, надо сохранять интригу. А пока ты ее сохраняешь, уставшие ждать элиты могут тебя самого стукнуть по лбу табакеркой… Это придает гибридам неустойчивость, которой они всеми силами стремятся избежать, но неизбежно именно к ней приходят.

- Является ли врожденным свойством гибридных режимов заигрывание с религией? Эдакая клерикальная конкиста?

Хороший вопрос! Это общемировая тенденция - такая легкая контрреформация после эпохи Просвещения, эпохи культа науки… Но, может быть, причина немножко в другом, если говорить о гибридах. Их уязвимое место - это действительно сомнительная легитимность. Они стремятся к легитимности процедурного типа, о чем мы уже говорили, но до конца не могут ее достигнуть, потому что выборы у них обычно фальсифицируются, пресса несвободна, открытой политической дискуссии не существует. Поэтому они всегда чувствуют, что сидят немного не по правде. И они стараются добирать эту недостающую легитимность другими инструментами.

Например, лидеры стараются изобразить из себя харизматиков. Вот покойный Чавес этим занимался. У нашего лидера это тоже немножко присутствует. «Да, выборы у меня странноватые, зато я, как гоголевский городничий говорил, в церковь хожу каждое воскресенье. Зато я наследник традиций!» В этом есть, конечно, элемент имитации, но это и попытка дополнить свою недостаточную легитимность.

Как гибридные государства реагируют на постиндустриальную эру, на переход к экономике человеческого капитала?

Понимаете, теория гибридных режимов говорит о политической структуре, не об экономической. Все гибриды предполагают рыночную экономику. Она может быть сильно огосударствлена, она может несправедливым образом раздаваться друзьям правителя, - но тем не менее там нет полного монополизма и полного огосударствления. Ни в одной из этих стран нет экономики советского типа. Соответственно, возможности для адаптации у них есть и в экономической сфере тоже. Но поскольку они боятся будущего и очень хотят остановить время (а еще лучше - чуть-чуть повернуть его назад), то им легче отстать. Особенно на этом повороте, который, похоже, сейчас делает человечество. Они стремятся контролировать конкуренцию и публичный дискурс, они не доверяют креативному классу, они предпочитают опираться на более отсталые и менее образованные слои. Кроме того, они все верят в игру с нулевой суммой…

- То есть не верят в общий выигрыш…

Да, не верят в win-win, в кооперацию - и они молятся на ресурсы. Все это готовит их на роль идеальной жертвы XXI века. Это не воображаемые опасения по поводу появления нового Сталина - меня раздражают эти разговоры, потому что они уводят от реальной угрозы. Наша реальная угроза - это стагнация и отставание. Реальная угроза - что нам в будущем достанется жалкое место, потому что даже привычная роль поставщика ресурсов опустится еще на пару позиций. Поскольку ресурсы уже не будут в такой степени нужны. И чем ловить Сталина под кроватью, лучше об этом задуматься. Хотя как об этом задумываться, если система вообще не настроена на то, чтобы говорить о будущем!

- И в завершение: какие варианты есть внутри этой системы у тех, кто думает?

Хорошая новость - гибриды не самоизолируются, не закрывают границы, так что уехать всегда можно. Еще одна хорошая новость - что мир стал единым и прозрачным, поэтому отъезд не имеет того рокового смысла, какой имела эмиграция в 1970-е или 1980-е. Как уехал, так и приехал. И как только трещины по льду пойдут, уехавшие вернутся обратно. Это не относится, вероятно, к генетикам и музыкантам - это скорее относится к людям, которые уехали по социальным и политическим причинам и имеют публичные амбиции. Их вообще полноценными эмигрантами назвать трудно.

Если же вы остаетесь, то надо понимать, что если лично по вам репрессии ударили, то вам не легче оттого, что они немассовы. Совет не участвовать в политической жизни, а изучать, скажем, санскрит, мне не представляется разумным. Во-первых, если вы так будете себя вести, то непонятно, почему вы не уезжаете в страну, где говорят на санскрите. Во-вторых, это не даст никаких преимуществ в момент трансформации режима. А моменты такие, как показывают научные данные, наступают.

Я бы посоветовала держаться в рамках закона. Это довольно трудно, потому что правовая база гибридных режимов обычно нестабильна, они любят менять законы и переписывать их под себя. Но я бы не советовала никому действовать революционными методами, уходить в подполье и создавать боевые организации. Вы сделаете себя уязвимым, а толку не добьетесь. Пользуйтесь преимуществами гибридов! Это преимущество в том, что они много что вынуждены имитировать. Они держатся за бумажку - и вы за бумажку. Они говорят: «Идите в суд!» - и вы идите в суд.

К тому же гибриды, в отличие от тоталитарных режимов, не уничтожают всякую гражданскую активность. Пользуйтесь этим, объединяйтесь друг с другом, участвуйте в деятельности общественных организаций. Да, борьба с НКО под видом борьбы с иностранными агентами - это такая привычная вещь. За последние 10 лет десятки режимов нашего типа приняли у себя подобное законодательство. Они все боятся гражданской кооперации, но совсем уничтожить они ее не могут. Они сами ее имитируют, создают то, что называется GONGO - Government-organized non-governmental organizations, «организованные государством негосударственные организации». Совсем ее убить у них нет ни возможностей, ни желания. Пользуйтесь этим! Совместная деятельность дает огромные силы.

На самом деле любая проблема в противодействии с государственной машиной решается при помощи трех ключей: организация, публичность и юридическая помощь. Если вы связаны с другими людьми, которые хотят того же, что и вы, и готовы действовать, если у вас есть выходы на СМИ (а в эпоху социальных сетей каждый сам себе СМИ), и если вы юрист или у вас есть возможность получить юридическую помощь, а ее тоже дают разные НКО, - то вы можете отстаивать свои права и интересы. Мировой опыт показывает, что это вполне реальные вещи.

Сейчас отовсюду раздаются призывы работать с молодежью. Как работать никто не знает, никто толком не понимает, кто они и что с ними делать. Как видите современную молодежь вы?

– Представление о том, что молодежь – это некие проводники в будущее, это наш завтрашний день, поэтому кто с ними договорится, тот и будет его бенефициаром и хозяином, кажется основанным на некоем неизменном ходе вещей. «Младенца ль милого ласкаю, уже я думаю: прости! Тебе я место уступаю: мне время тлеть, тебе цвести». Но на нынешнем историческом этапе эти, казалось бы, неизбывные истины подвергаются некоторой коррекции.

Во-первых, наша с вами молодежная страта немногочисленна : это плоды демографической ямы 90-х, которая, в свою очередь, стала наследницей предыдущего демографического провала Второй мировой войны. Если посмотреть на нашу с вами демографическую пирамиду, видны эти повторяющиеся вмятины – нерожденные дети мертвецов. Эта яма немножко сглаживается с течением лет и будет сглаживаться дальше, если дальнейшее наше историческое развитие пойдет без катастроф, но она есть.

Во-вторых, представление о смене поколений устаревает. Есть такой рассказ у Киплинга – «Поправка маленького Тода», из сборника его рассказов о британской Индии. Там рассказывается, как маленький мальчик забрел на заседание законодательного совета, на котором сидели британские администраторы, и там пересказал возражения своих индийских слуг по поводу предлагаемого закона, по которому перезаключать договор об аренде земли нужно было бы каждые пять лет, а не пятнадцать, как раньше. Они сводились к тому, что за пятнадцать лет человек вырастает и становится мужчиной, рождается его сын, еще через пятнадцать этот сын уже мужчина, а отец уже умер, земля переходит к следующему работнику. Если перезаключать эти договоры каждые пять лет, это лишние расходы, суматоха и деньги на всякие пошлины и марки.

В традиционном обществе с низкой продолжительностью жизни смена поколений идет очень быстро – как раз за пятнадцать лет. Мы сейчас ориентируемся на двадцать пять лет, но ситуация меняется: продолжительность жизни увеличивается. Соответственно, увеличивается и срок активной жизни, и удлиняется срок детства. Я не ожидаю, что через двадцать пять лет у меня наступит «возраст дожития», как это деликатно называет наш Пенсионный фонд, а мои дети будут отцами и матерями семейств и главами домохозяйств. Скорее всего, я еще буду работать, а мои дети, возможно, будут еще учиться, искать себя, у них не будет своих семей и детей, они еще будут молодежью.

Смена поколений очень сильно замедлилась, поэтому с чисто прикладной точки зрения, если вы хотите политической власти и влияния, то работайте с теми, кому сорок. Их много – это многочисленное поколение, дети «советских беби-бумеров», они уже давно на социальной сцене и еще лет тридцать будут проявлять себя социально, экономически и политически. С этой точки зрения молодежь можно немножко оставить в покое.

Тем не менее, пока мы еще не достигли биологического бессмертия, которое нам недавно обещал Алексей Кудрин в перспективе 10-12 лет (правда, не в России), поколения все же сменяются. В связи с этим мне представляется важным исследование поколенческих ценностей, семейных отношений, стилей родительства, гендерного контракта и его изменений.

Когда говоришь «молодежь», «дети и родители», каждый подразумевает что-то свое. Надо помнить, что поколение миллениалов – это поколение людей, которые достигли возраста ранней социальной зрелости к смене тысячелетий. То есть это родившиеся в конце 70-х – начале 80-х. Нынешние двадцатилетние – это так называемые центениалы, поколение Z. Эти два поколения различаются между собой. Полезно помнить, что у человека 45 лет вполне может быть ребенок 20 лет – это социальная норма. Поэтому когда мы говорим «родители», мы не должны представлять себе каких-то седобородых старцев, мы должны представлять молодых людей в диапазоне от 40 до 55.

У нас сейчас на социальной сцене активны три демографические страты. Люди 60+, рожденные в 50-е, занимают верхние этажи управленческой пирамиды. Есть поколение 40+, их дети, родившиеся в 70-е. И есть поколение новое, которое и есть молодежь, – родившиеся в 90-е и позже.

С точки зрения демографической статистики, наш с вами демографический провал завершается в середине двухтысячных. С 2004-го по 2014-й была зафиксирована высокая рождаемость. Это два кирпича в основании нашей демографической пирамиды: те, кому сейчас от 0 до 5, и те, кому от 5 до 10. Когда они войдут в возраст социальной активности, наступит интересный момент. Хотите готовиться к политическому будущему – сейчас работайте с сорокалетними, а через десять лет ждите новых двадцатилетних, их будет много.

Хотите власти – имейте организацию

Поскольку я политолог, любая демография и поколенческие ценности меня волнуют ровно настолько, насколько они отражаются на политических процессах и политическом поведении. Когда мы говорим о политических процессах, простое количество участников мало что значит. Оно важно, потому что это голосующие, но с точки зрения влияния на политические процессы важно не число по головам, а организованность структуры. Это общий закон, он не знает исключений.

Неорганизованное в политическом пространстве не имеет субъектности, организованное – имеет. Власть всегда принадлежит организованному меньшинству, но вместо того, чтобы печалиться по поводу этого железного закона об олигархии (как это научно называется), организуйтесь – и вы тоже будете обладать властью. Власть – это не игла в яйце, ее можно найти во всех социальных отношениях: в семье, в экономическом обмене, в производстве, в творчестве. Хотите власти – имейте организацию.

Молодежи сейчас мало, но, учитывая, что наша цивилизация в целом ценит молодость и считает будущее и новое позитивными маркерами, участие молодых людей в любом процессе повышает его цену. Если у вас одни пенсионеры, считается, что вы люди вчерашнего дня.

На самом же деле, если вы сможете привлечь голоса и энергию пенсионеров, они долго будут служить вам политическим топливом для ваших нужд и целей. С молодежью как в игре «Скрабл»: если вы на эту клеточку сумели поставить свою буковку, то цена вашего хода сразу удесятеряется.

Где конфликт поколений?

– На телевидении с некоторой паникой понимают, что потеряли молодежную аудиторию, она ушла в неконтролируемые социальные сети. При этом достаточно много молодых людей вообще отказывается от социальных сетей и активного присутствия в интернете. Где они, что они собой представляют?

– Насчет паники вы очень правы. Она охватывает административную машину – может быть, пока еще с недостаточной силой. Когда они или от их имени говорят о том, что «мы потеряли молодежь», что молодежь телевизор не смотрит, или власти не уважает, или на выборы не ходит, или еще что-то не хочет делать, то молодежь тут – всего лишь псевдоним завтрашнего дня. На самом деле у тех, кто наверху, проблемы не с молодежью, а со следующим поколением, с их собственными детьми. Они их по привычке называют молодежью, а это уже давно не молодежь. Это люди в расцвете социальной зрелости, и они лишены доступа к принятию решений и политическому представительству.

Сейчас все исследования межпоколенческих и семейных отношений показывают нам интересную штуку. Мы привыкли считать, что конфликт поколений – это вещь, заложенная природой: дети всегда бунтуют против отцов, так уж жизнь устроена. Мы не отдаем себе отчет, до какой степени конкретные социально-исторические условия способны этот конфликт сгладить либо обострить.

Мы сейчас будем говорить об очень больших общностях, внутри которых будет много исключений, поэтому не пытайтесь проецировать эти наблюдения на свои семьи. В самом общем виде картина у нас следующая: люди, рожденные в 50-е годы, очень своеобразно выполняли свою супружескую и родительскую функцию. У этого поколения есть свои особые характеристики: самый высокий уровень разводов и абортов, модус этих разводов и модель последующих отношений родителей с детьми, специфическое половое поведение 70-х и 80-х. Мы не будем сейчас уходить в причины, не будем никого обвинять или оправдывать, просто зафиксируем этот социологический факт.

Это поколение было сорокалетним к распаду Советского Союза. Часть восприняла это событие как величайшую политическую катастрофу, часть – как открывшееся великое окно возможностей, это сейчас не важно.

Важно, что этика и эстетика, политика и экономика 90-х во многом стала отражением представлений о жизни именно этого поколения. Когда говорят, что мы выстроили капитализм по книге «Незнайка на луне» и по карикатурам в «Крокодиле», изображавшим капиталистическое общество, а отношения Церкви и государства – по перевернутым наоборот атеистическим брошюркам и Емельяну Ярославскому, надо иметь в виду, что те, кто строил все это, были воспитаны по-советски.

Поколение рожденных в 50-е – вершина советского воспитания, они прошли полный курс идеологической индоктринации: от детского сада до высшей школы. Война отрезала навсегда память о прежней России, просто физически убив всех, кто мог что-то помнить, а послевоенное поколение стало продуктом советской власти.

Их отношения с собственными детьми, скажем аккуратно, склонны быть сложными. Именно в их случае конфликт поколений проявляется максимально остро. Женщины и в меньшей степени мужчины 40+ – основная клиентура психологов и психотерапевтов, и их запрос – исправление травм из детства. В поколении рожденных в 50-е конфликт поколений проявляется максимально остро.

Обычно считается, что сорока- и пятидесятилетние обижены отсутствием социального и карьерного лифта: дети генералов доросли до генеральских должностей, а ротации не происходит. Но дело не только в этом. Очень часто конфликт обусловлен тем, что дети представителей этого поколения выросли в распавшихся семьях с очень специфическими отношениями между отцом и матерью. Это дети советских женщин с их особым пониманием своей роли, своих обязанностей, своих прав по отношению к детям и по отношению к действующим и бывшим мужьям.

Дети поколения 50-х уже имеют своих детей. И вот между «детьми» и «внуками» нет конфликта поколений, причем такая тенденция фиксируется не только у нас. Сглаживание конфликта поколений между центениалами и их родителями отмечается везде. Это довольно уникальная ситуация с точки зрения антропологии.

Более всего обращает на себя внимание исследователей то, что дети и родители говорят друг о друге с нежностью и уважением. Это кажется самой естественной вещью на свете – кто ж не любит своих детей, и родителей принято тоже любить. Но в середине двухтысячных картина была противоположной.

Я помню, как читала в «Живом журнале» закрытые женские сообщества, и у меня возникло жуткое чувство, что я среди своих сверстников, а на тот момент мне было тридцать, вообще одна со своими родителями разговариваю. Люди находились в жутком конфликте с родителями: либо вообще не общались, либо ненавидели друг друга, даже телефонные разговоры заканчивались истериками, слезами и бросанием трубки. Мне лично это было дико.

Типичная история.

– А вот на следующем демографическом шаге это уже не типичная история. Большинство поколенческих исследований носит маркетинговый характер: понятно, что компании хотят узнать, кому как продавать товары и услуги. Тем не менее, мы, политологи, можем много интересного из них извлечь. В исследовании, которое недавно проводилось для Сбербанка, есть такой интересный момент: одна из немногих претензий, которую дети предъявляют к родителям – что те не говорят, как жить, не дают установок.

Слишком много было установок у них самих?

– Может, было много установок у них самих, может быть, они чувствуют, что время меняется слишком быстро. Родители, в свою очередь, говорят: «Я не знаю, как надо, они, может быть, лучше моего знают». Обычно о том, что впервые в истории человечества следующее поколение знает больше, чем предыдущее, пишут в исследованиях, касающихся цифровой грамотности и сетевого бытия. Обучение идет в обратном порядке, и это, мягко говоря, взрыв мозга, потому что вся наша культура построена на том, что предшествующее поколение передает свой опыт следующему.

Такая передача опыта характерна прежде всего для аграрного общества, где инноваций практически нет, а опыт важнее, чем творчество. После того, как начались последовательные волны промышленных революций, а великие географические открытия расширили горизонты человечества, уже возникала ситуация, когда следующее поколение лучше ориентируется в изменившихся условиях, чем предыдущее.

Но обычно за то время, когда условия жизни менялись, эти новые поколения сами успевали стать взрослыми и родителями. На таком коротком временном отрезке это явление наблюдается впервые. Это очень интересный, новый и мало на что похожий феномен.

Невротическое стремление быстро-быстро напихать в ребенка умения и навыки с тем, чтобы он был подготовлен к жизни, сменилось ощущением, что нельзя в него ничего инсталлировать, потому что мы не знаем, как изменится мир завтра.

Идея, что до 21 года ты учишься всему, что ты должен знать, а дальше только работаешь на этом топливе, уже выглядит утопичной.

С одной стороны, время бежит быстро, а с другой – торопиться некуда: все понимают, что ты будешь обучаться бесконечно, повышая квалификацию или получая новую специальность. От этого понимания возникает желание не тратить совместные с ребенком годы жизни на то, чтобы впихивать в него насильно, как в гуся для фуа-гра, ценные знания и в процессе портить отношения, а лучше дать ему запас любви, чувство собственной ценности и принятия, которые останутся с ним.

Я сейчас не говорю, что это рациональная или выигрышная стратегия: те, кто получил лучшее образование в молодости, все равно имеют преимущество – не оттого, что они узнали про таблицу Менделеева, а оттого, что у них больше нейронных связей в голове образовалось в процессе узнавания таблицы Менделеева, поэтому их мозг лучше приспособлен к дальнейшему обучению.

Я сейчас говорю лишь о том, что у людей возникает некое ощущение, что главное – это все-таки отношения, любовь. Вот я даю своему ребенку уверенность, принятие – а за этим стоит ощущение, что дрессировка, которую давали родители предыдущего поколения, уже не выглядит такой уж ценной.

Когда находящиеся при власти говорят о молодежи, которую они упустили, они говорят не о молодежи. Они упустили своих детей. Эта формулировка справедлива для значительного количества людей этого возраста, но слава Богу, не для всех – человеческая природа берет свое.

Упущенные дети – это кто?

– Это те, кого родили люди поколения 50-х годов.

Если мы говорим о молодежных протестах, то это не протесты двадцатилетних против своих родителей. Поколения двадцатилетних детей и их родителей объединяют общие ценности, основная из которых – это справедливость. Их протест проявляется по-разному, в зависимости от возраста.

Сорокалетние и более старшие склоняются к протестам законными методами, и это хорошо и эффективно. Эти люди записываются наблюдателями, подают заявления в суды, пишут жалобы, искусно стравливают одно ведомство с другим, чтобы получить желаемое, организуют структуры, которые защищают права заключенных, женщин, детей, больных, кого угодно. Они успешны в этой деятельности. Протест «внуков» в силу их возраста носит более хаотичный характер.

Вопреки тому, что любят говорить о русском народе, уровень толерантности к насилию у нас низкий, в том числе к государственному насилию. У нас, может, и любят поговорить про Сталина, которого на вас нет, но как только начинаются реальные проявления государственного насилия, это мало кому нравится. Еще точнее, те, кому это не нравится, гораздо организованней и артикулированней тех, кому норм.

Асексуальность – новый тренд, а уровень насилия снижается

– Вы начали говорить про мораль и ценности молодежи. Складывается противоречивая картина: с одной стороны, молодые люди снимают всякие жестокости на видео и выкладывают в YouTube, с другой – очень много новостей, где какой-то старшеклассник кого-то спас.

– Часто цитируют надпись внутри одной из египетских пирамид, что нынешняя молодежь не хочет работать, не чтит богов, старших, хочет только развлекаться и так далее. Переживание по поводу низкого морального облика молодежи и вообще по поводу большего по сравнению со вчерашним разврата – это тоже один из традиционных социальных механизмов передачи опыта. Интересно, что в нынешний исторический момент это утверждение наиболее далеко от истины.

Все те данные, которые мы имеем, причем как американские, так и российские, говорят о том, что все дальше и дальше отодвигается вовлечение молодежи в те практики, которые раньше считались маркерами взросления.

Люди все позже пробуют алкоголь, все позже начинают курить или не начинают вовсе, все позже начинают половую жизнь. Поколение Z вообще гораздо меньше интересуется сексуальной тематикой, чем любое предыдущее. Асексуальность – это новый тренд, и он будет только развиваться.

Все исследования свидетельствуют о том, что нынешняя молодежь – самое правильное из всех поколений, какие только можно себе представить.

Все исследования свидетельствуют о том, что нынешняя молодежь – самое правильное из всех поколений, какие только можно себе представить.

Вырастая, люди забывали об этом, понятие о насилии было размыто, толерантность к насилию была гораздо выше. Считалось, что все мальчики дерутся, это нормально и правильно. Сейчас кто-нибудь так считает? – нет. Следует ли из этого, что мальчики больше никогда не дерутся? Нет, не следует, но отношение изменилось, и это влияет на поведение.

Мы присутствуем при очень медленном отмирании инициационных практик, которые предполагали, что в возрасте полового созревания весь пул молодежи подвергается чему-то, что переживают не все. Кто-то отсеялся, а тот, кто пережил, – уже с боевыми шрамами входит в состав племени и считается полноценным охотником, добытчиком, имеет право на секс, имущество и автономию. Эти практики очень глубоко укоренены в нашем сознании, это сюжет значительного числа волшебных сказок и большинства художественных произведений о взрослении.

Сейчас для того, чтобы стать мужчиной, ты уже не должен убить себе подобного. Постепенно уходят и ситуации, когда тебя должны бить, и ты должен это пережить, или ты должен побить кого-то и, соответственно, пережить это. Мы сейчас не будем говорить, какими будут последствия и чем эти практики будут заменены, просто фиксируем этот факт.

Толерантность к насилию у нас все ниже и ниже, поэтому факты, на которые раньше никто не обращал внимания, становятся предметом обсуждения и возмущения – к тому же благодаря техническим средствам все запечатлевается и публикуется.

Возникает впечатление, что в мире чудовищная жестокость – девочки побили другую девочку и выложили съемку в интернет. Да назовите мне класс, в котором девочки или мальчики не били другую девочку или мальчика! Просто телефонов с камерой раньше ни у кого не было.

Мы еще не осознаем масштабы снижения насилия, мы просто его наблюдаем. Вообще, глобальное снижение преступности, great crime drop – это одна из загадок, над которыми бьются представители всех общественных наук сразу.

Почему люди перестали совершать преступления? Среди попыток объяснения этого феномена есть довольно экзотические, типа улучшения качества бензина и снижения количества свинца в выхлопах. Свинец, как известно, повышает агрессию.

Американская версия: поколение преступников просто не родилось, потому что тридцать лет назад неблагополучным слоям стала доступна контрацепция.

Не улучшилась статистика только по двум видам преступлений: это киберпреступления и почему-то кражи мобильных телефонов. Количество случаев уличного хулиганства снизилось очень сильно, и одна из причин, которые называют, – компьютерные игры.

Компьютерные игры вообще нас всех спасут: это и новые рабочие места, и симулякры войны для молодых людей. Как социуму обойтись без войны, когда для всех предыдущих поколений человечества это было основное занятие элиты, способ решения политических конфликтов, способ экономического продвижения? Чем заниматься политической верхушке, если войну отменили?

Исследования показывают, что молодежь все больше интересуется едой. Вы обратили внимание, как много мальчиков и девочек учатся готовить?

Если раньше «в кулинарный техникум пойдешь» было страшным проклятием, то сейчас наоборот.

– Это чудная, творческая и очень востребованная профессия, где нас еще некоторое время не заменят роботы. Сейчас, выбирая профессию, нужно задавать себе вопрос: это может делать робот? Если может – не занимайтесь этим.

Шеф-повар – это же вообще одна из самых высокооплачиваемых профессий!

– Это и новые звезды. Никто больше не хочет смотреть на рок-музыкантов, употребляющих наркотики. Все хотят смотреть на Джейми Оливера, который что-то там готовит в обществе своих пятерых детей.

Отсутствие мотивации будет социальным преимуществом

– При этом часто говорят, что у современной молодежи достаточно низкий уровень мотивации. Я сама чувствую, что не могу сказать своим детям: «Учись хорошо – будет у тебя все хорошо, иначе в дворники пойдешь». Я понимаю, что сегодня люди, которые не закончили даже десять классов, совершенно прекрасно устроены и все у них хорошо.

– Отсутствие мотивации может стать прекрасным и очень актуальным свойством для того поколения, которому предстоит жить в экономике пост-дефицита и, возможно, пост-труда.

Представьте себе, что автоматизация производства дала нам чрезвычайное удешевление всего того, за что убивались люди предыдущих поколений: мебели, бытовой техники, автомобилей, одежды, других материальных предметов. Что, действительно, после экономики владения наступает экономика пользования. Что наши потомки будут смотреть на нас с нежной жалостью за то, что мы стремились приобрести куски собственности и таскали их за собой.

Может быть, утром дроны по предварительному заказу будут доставлять к их двери капсулы с одеждой, а вечером забирать. У них не будет собственности, жилье будет съемным. Объективно они будут беднее нас, но их уровень жизни будет выше.

Это кажется парадоксом, пока мы не попробуем оглянуться на какой-нибудь предыдущий исторический период и взять для удобства сравнения уровень потребления и уровень жизни тогдашней элиты.

У аристократии были бриллиантовые тиары и дворцы, которых у нас нет, но при этом у них не было возможности лечить зубы, они умирали рано и страшной смертью, их дети мерли, как мухи, они физически безумно страдали, жили в дискомфорте, в холодных помещениях со сквозняками, у них не было канализации и водопровода, им было трудно помыться – в общем, каким бы ты ни был выдающимся царем, графом или герцогом, с нашей точки зрения твой уровень жизни и комфорта был чудовищно низким.

Если этот процесс будет продолжаться, если он даст те результаты, которые нам сейчас описывают футурологи экономической направленности, то отсутствие мотивации бежать за ускользающим долларом или убегающим рублем с целью поймать его и обеспечить себе жизнь – это будет очень хорошо.

Отсутствие такой мотивации будет социальным преимуществом, потому что человеку будет нужна мотивация иного типа: мотивация к самореализации, к проявлению своей уникальности, того в себе, что не может заменить робот.

Труд в нашем сегодняшнем представлении станет никому не нужен, потому что от твоего труда только экологическая ситуация ухудшится, а вот от твоего творчества будет прибавочная стоимость и дальнейший прогресс человечества.

Если выражаться менее возвышенно, отсутствие мотивации – чрезвычайно ценное качество для людей, которым предстоит жить в обществе, где их работа не нужна. Для того чтобы они не чувствовали себя выброшенными из социума и никому не нужными, у них должна быть другая психология, другое устройство головы. Они не должны считать получение фишечек целью своих усилий. Они должны спокойно относиться к осязаемым достижениям, к должностям, наградам, деньгам – собственно говоря, к внешним признакам статуса.

Мы видим, как человечество потихонечку к этому идет. Смотреть всегда надо на Первый мир и его передовые отряды, потому что они задают нормы, которые потом будут всеобщими. Там мы видим пятьдесят серых толстовок Цукерберга, скандинавизацию поведения элит, показную скромность и смерть того демонстративного потребления, которое в свое время принесла с собой буржуазия, когда стала правящим классом.

Хороший человек – это профессия

Возникает проблема нового века: как и чем занять людей, чья работа не нужна. Кажется, что жизнь с гарантированным гражданским доходом, когда работать не надо, будет прекрасной мечтой, но на самом деле человек от этого болеет и умирает. Исследования показывают, что у лишившихся работы людей процесс саморазрушения начинается гораздо раньше, чем наступает материальная нужда.

Человек должен быть включен в социум, ему нужно признание, ему нужно ощущать себя важным и полезным, делающим нечто ценное, ему нужны смыслы. Если дать ему деньги и сказать: «А теперь иди и ничего не делай», – он начнет болеть, чахнуть и разрушать себя.

Известный экономист Роберт Скидельски, бывший член британского парламента, сказал следующее: одна из задач новой эпохи – научить всех жить так, как раньше жила только аристократия, и при этом не сойти с ума. Кажется, что это вообще не проблема, но на самом деле это очень большая проблема.

Решаться она будет тем поколением, которое, слава Богу, равнодушно к цацкам и понтам, которое наконец скинет это ярмо со своей души, которое сейчас уже говорит, что главная ценность – это семья, что создание семьи – это большее достижение, чем карьерный успех, что главное – это отношения, которое ценит коммуникативные навыки.

Это очень правильно, потому что пресловутая эффективность есть у робота, человеку она становится нужна все меньше и меньше. Помните, было такое советское выражение: «хороший человек – это не профессия»?

Сейчас мы приходим к обществу, в котором другой профессии нет: есть только профессия хорошего человека, а все остальные могут быть автоматизированы.

Сейчас мы приходим к обществу, в котором другой профессии нет: есть только профессия хорошего человека, а все остальные могут быть автоматизированы.

От человека требуется коммуницировать с другими людьми, создавать и поддерживать отношения, организовывать людей. На первый план выходят менеджерские качества, но не в смысле выжать максимум из работника, а в смысле поддержать совместную работу, сделать ее радостной и удовлетворяющей тех, кто в нее вовлечен.

Это становится чрезвычайно ценным, и в этом смысле новое поколение выглядит очень перспективно. Вообще, кто общается с двадцатилетними, тот в большом восторге от них, я как преподаватель могу это подтвердить.

Ценность семьи будет только расти

Происходит размывание границ «мужского» рабочего и «женского» домашнего пространства. Повышение ценности семьи и семейных отношений привело к тому, что женщины не захотели бросать своих детей, но и работу бросать тоже не захотели. Великая дилемма «семья или работа» осталась в ХХ веке: это проблема для промышленной экономики, когда твоя работа состоит в том, что ты либо сидишь в конторе, либо стоишь на заводе. Все больше и больше людей работает дома и выезжает на совещания, только чтобы хоть как-то обувь на каблуках выгулять.

Ценность семьи будет только расти, потому что люди все больше и больше живут дома. Удаленная работа и развитие доставки возвращает нас по домам. В ХХ веке человек у себя дома, считай, и не бывал: он утром на завод выезжал, с завода вечером приезжал, в отпуск ездил в санаторий, детей на три месяца отправлял в пионерский лагерь, и посмотреть, кто у него в квартире живет, возможности толком не было. Это, с одной стороны, укрепляло семейные отношения, с другой – уничтожало их, кому как повезет.

Сейчас люди живут дома и ставят свои отношения с домашними на первый план. Это чем-то напоминает традиционное общество: изба и прялка, только вместо прялки у нас компьютер. А когда вертикальные фермы появятся и накормят наши города, поселения будут становиться все более и более автономными.

Мы еще увидим какие-нибудь деревни староверов или поселок художников, которые вообще ни в чем не нуждаются: у них есть солнечная батарея на крыше, от которой они получают электричество, они провертели себе скважину, оттуда получают воду.

У них стоят вертикальные фермы, на которых они выращивают себе еду, к ним прилетает дрон и приносит все, что им нужно, не говоря уже о том, что они это могут напечатать на 3D-принтере, который стоит прямо тут же. Жизнь в городах изменится очень сильно.

Очередь выстраивается за окситоцином

– При этом нет ли такого чувства, что очереди за айфонами и какими-то особыми кроссовками – это свидетельство повышенной потребности в маркерах своего социального статуса?

– Это квест, это приключение. Раньше человек старался избежать физического труда, потому что это было проклятье и удел нижестоящих. Чем выше ты взбирался по социальной лестнице, тем меньше ты работал физически и тем больше ты ел жирной еды. Богатый от бедного отличался очень просто: у богатого были длинные ногти, белые ручки и специальная одежда, которая показывала, что он не работает, и уж в совсем традиционных обществах ориентального типа у него был еще большой живот (может позволить себе есть много жирного мяса!).

Сейчас все перевернулось: бедный – толстый, богатый – худой. Мы специально бегаем и прыгаем, занимаемся физическим трудом и поднимаем тяжести для того, чтобы быть здоровыми. Точно так же стояние в очереди, которое было проклятием для советского человека, высасывало его кровь, делало его агрессивным и вообще уничтожало его жизнь, теперь становится прекрасным увлечением. Смотрите, мы стоим все вместе, у нас adventure, люди покупают специальные билеты на то, чтобы им квест устроили.

– Я несколько раз слышала от людей, которые организуют квесты, что от них у молодежи какая-то наркотическая зависимость.

– Несмотря на онлайн, несмотря на прославленные мной компьютерные игры, природа человека не изменилась: человек – социальное животное, он нуждается во взаимодействии с себе подобными. Это взаимодействие в онлайне не хуже, чем в офлайне, но человек хочет взаимодействовать и в реальном мире. Квесты дают не столько адреналин, сколько командность.

Кстати говоря, это ровно то, зачем люди ходят в благотворительность, некоммерческие организации, политический активизм. Многие думают, что люди ходят туда жертвовать собой, – это очень опасное заблуждение. С теми, кто пришел с такими представлениями в благотворительность, будут происходить плохие вещи.

Надо понимать, что люди приходят туда за окситоцином – гормоном счастья, который вырабатывается при успешной совместной деятельности. Тот, кто попробовал сладкий вкус успеха во взаимодействии с другими, будет за этим приходить еще и еще.

Вообще-то этот опыт должна давать человеку школа. «Я не знал, я узнал, и теперь у меня получилось». Если бы кто-то обладал достаточным педагогическим талантом для того, чтобы воспроизводить этот опыт для учеников, то дети обожали бы школу. Делать то, что получается – это большое удовольствие.

Принудительная публичность – новый инструмент давления

– У нас получилась совершенно идеальная картина современной молодежи. Какие у них есть проблемы, темные стороны?

– Люди, которые смотрят на происходящие социокультурные процессы недоброжелательными глазами, называют формирующуюся культуру культурой слабости – в противоположность культуре силы, которая была до того.

Что плохого можно сказать об этой культуре слабости? Она фетишизирует жертву и тем самым побуждает людей объявлять себя жертвами для того, чтобы получить привилегии. Снижая общий уровень насилия, в особенности физического, она вырабатывает новые формы насилия, первой из которых я бы назвала принудительную публичность.

Есть в соответствующем сообществе термин «аутинг». Есть каминг-аут, когда ты рассказываешь о себе, а есть аутинг, когда я рассказываю, что ты такой-то. Это инструмент давления новой эпохи. Парадоксально, но, как и в традиционном обществе, в обществе новом все оказываются завязаны на репутации. Все живут на виду, всё открыто, записано и может быть опубликовано, данные доступны не только государствам и корпорациям, но и гражданам.

– О тебе все известно, начиная с момента, когда мама пришла в мамское сообщество и сказала: «Что-то у нас сегодня с памперсом проблемы».

– Да, совершенно верно, и твоя фотография с памперсом и без из глобальной сети не исчезнет никогда и будет преследовать тебя по жизни. Соответственно, репутация – это все, и крах репутации закрывает человеку все его социально-профессиональные перспективы. Он не может сказать: «Да, предположим, я сволочь и плохо поступал, но зато я профессионал».

Никому не нужен твой профессионализм. Ты продаешь некий продукт, центральный элемент которого – твоя личность. Если твоя личность вызывает отвращение и отторжение, то нельзя сказать: «Да, я дал женщине по заднице, но я хороший актер». Неважно, какой ты актер, люди приходят посмотреть на тебя в фильме, и они должны к тебе хорошо относиться. Если они к тебе плохо относятся – они не пойдут на фильм с тобой, есть много других фильмов с хорошими людьми.

Викторианское какое-то отношение.

– Мы уже упоминали о специфическом отношении к сфере сексуального среди молодого поколения. Надо признать, что мы с вами въезжаем на полной скорости в культуру, относящуюся к сексуальности если не негативно, то подозрительно.

Для всех нас было бы лучше, если бы нормы задавала старая добрая развратная Европа, но в современном мире они задаются Америкой, а Америка – страна пуританская. Они буквально несколько десятилетий, с конца 60-х, прожили в ситуации, когда секс считался чем-то скорее хорошим, чем плохим, и, видимо, им не понравилось.

Сейчас мы видим, как американское общество с очень большим удовольствием возвращается в парадигму, в которой секс – это плохо. Когда они были пуританами, то говорили, что это грешно, теперь они говорят, что это опасно. Сексуальное общение становится опасным с разных сторон: во-первых, ты никогда не можешь быть уверен, что твое поведение не будет признано насилием, а во-вторых, ты открываешься другому человеку и не знаешь, как он себя поведет. Это было всегда, но сейчас эти риски превышают выгоды.

При доступности технологических средств для решения этой проблемы, следующим поколениям идея, что ради того, чтобы получить оргазм, нужно связываться с целым другим человеком, будет казаться дикой. Отношения они будут ценить, конечно, но секс будут ценить меньше. Так что целомудрие и воздержанность – это, похоже, наше все.

Отстаивать права будут менее агрессивно, но более настойчиво

Новое поколение, возможно, будет по нашим понятиям более трусливым. Пойти против общества будет с каждым следующим поколением все более и более сложным делом. У людей есть потребность жертвовать собой, но когда на твоих общественных отношениях так много всего завязано, а уровень комфорта так велик, эта потребность реже будет реализована.

Если смотреть с политической точки зрения, отсутствие ярко выраженной мотивации на победу и достижения и социальная конформность может сделать из них более пассивных граждан. Но, с другой стороны, идея о максимальной ценности самовыражения и самореализации, а не накопления материального, будет работать против той тенденции, которую я описала: человека, который полностью завязан на материальный стимул, еще проще сделать конформистом. Человек, который понимает, что он не будет социально успешен, если не будет развивать свою личность, и который свою личность ценит выше всего, будет менее агрессивен, но будет тщательнее стеречь свои границы и с большей настойчивостью отстаивать свои права.

Сейчас по сети гуляет текст про молодую девушку, которую с ребенком положили в больницу, и она там устроила борьбу за свои права, потому что ей не понравилось, как с ней обращаются.

Дети, рожденные в 90-е, стали родителями, и они не считают унижающее и агрессивное отношение нормальным. Самое главное – что норма меняется.

Нормой может быть все что угодно: жертвоприношение первенцев, ритуальное убийство, храмовая проституция, геноцид. Человек – настолько пластичное существо, что в зависимости от условий и общественных установок он может вести себя как ангел, а может как последняя сволочь (причем один и тот же человек). В психологических экспериментах вроде стэнфордского, когда людей переодевают в заключенных и охранников, они начинают несусветные вещи вытворять. Когда нужно током бить за неправильный ответ того, кого ты не видишь, люди доходят до смертельного, как они считают, напряжения.

Обычно эти результаты интерпретируют в том духе, что каждый человек в душе – кровожадное животное. Ничего подобного. На самом деле эти эксперименты говорят о том, что человек бесконечно адаптивен, он соблюдает правила. Это наша психическая норма: какие правила – такие и мы, поэтому изменение правил, изменение понятий о приемлемом чрезвычайно важно. Если мы видим снижение толерантности к насилию во всех его формах, общий тренд не может не радовать.

Сейчас есть большой голод по милитаристским ценностям.

– Прошу прощения, что я сразу к выводам перехожу, но, как мы видим на основании данных исследований, это, похоже, последние гастроли поколения 60+.

Главный принцип родительства, как и в медицине – не навреди

– Моим детям 9 лет, 5 с половиной и 2 года и 3 месяца. Я еще на том идиллическом этапе, когда от меня не требуется особых родительских подвигов по налаживанию отношений. В этом смысле хорошо иметь много детей, потому что, по прекрасной формуле, принадлежащей моему мужу, все счастливые семьи похожи на ферму или небольшой питомник.

Когда больше двух детей – это уже не совсем частная жизнь, это такое предприятие. Производственный элемент во многом упрощает жизнь, отношения выстраиваются вокруг этой производственной необходимости довольно здоровым образом: вас много, я одна, есть какие-то вещи, которые надо сделать, это все понимают и в это встраиваются.

При том, что это усложняет жизнь логистически, морально это ее упрощает. Я думаю, что люди, запертые наедине со своим единственным ребеночком, которые думают, как его развивать, как с ним общаться, как не подавить его личность, может быть, в какой-то мере ведут более сложную и нервную жизнь.

– Какие главные навыки и компетенции вы хотели бы заложить детям? Александр Архангельский , – это умение действовать по-новому и искать выход в новых ситуациях. Мы не можем дать полный объем знаний, потому что они станут другими, а вот адаптироваться к переменам научить можно.

– Как человек, выросший в семействе педагогов, могу сказать такую вещь: педагоги сами не очень верят в воспитание и очень верят в наследственность. Воспитание – это здорово, но ребенок вырастает похожим на своих родителей. Мы просто живем все вместе и, поскольку это мои дети от моего мужа, то я не считаю, что они как-то принципиально глупее меня. Они сами свои скилы прокачают.

Я совершенно не верю в идею конкуренции между людьми: люди разные и хотят разного, поэтому, если они конкурируют за один объект, скорее всего, одному из них этот объект не нужен, просто он еще не догадался об этом. У Гофмана есть такая новелла, называется «Выбор невесты». У невесты было три жениха, они все хотели на ней жениться. Потом пришла фея и предложила каждому выполнить его желание.

У читателя возникает вопрос: как же так, они все хотят эту невесту?! В результате один из них получает невесту, второй – кошелек, в котором никогда не кончаются деньги, а третий – книжку, которая по желанию превращается в любые книжки (киндл!). Один из них любил девушку, другому нужны были деньги, а третьему хотелось бесконечную библиотеку, при этом они все конкурировали за эту самую невесту. Я думаю, что эта ложная невеста и является драйвером ложной идеи конкуренции.

Я не верю в то, что можно деток так выдрессировать, чтобы они были конкурентоспособны. Как показывает практика, главными препятствиями на пути жизненного успеха и счастья становятся не недостаток навыков и знаний – они приобретаемы, а собственные психологические депривации. Нам мешают тревожность, страхи, обсессивно-компульсивное расстройство, склонность к анорексии и прочее подобное. Если всего этого не будет, если человек будет психологически достаточно здоров и благополучен, то он добьется всего, чего хочет.

Мне кажется, я для детей уже все основное сделала: сама их родила от наилучшего возможного отца, выращиваю в благополучной семье, где никто их не обижает, а если кто-то пытается обижать снаружи, то я не поощряю такого поведения. Вот, собственно говоря, и все. Принцип «не навреди» что в медицине, что в родительстве базовый.

Навредить легко – человечество накопило большой опыт в этом деле, а вот дать ребенку вырасти цельным, по дороге не натыкав ему пальцем в чувствительные места, затруднительно. Я бы скорее следила за собой в этом отношении. Как нынче говорят, как бы вы себя ни вели, ваши дети найдут, на что пожаловаться своему терапевту. Я принимаю этот факт – пусть они жалуются терапевту. У кого мама была дома, те будут жаловаться, что мама все время присутствовала и нависала. У кого мама работала – что ее не было и не хватало…

Иногда боишься, что накричишь на детей, а они с этого начнут свой поход к психотерапевту через 15 лет.

– Как говорил Аристотель, берегите слезы своих детей, чтобы они могли пролить их на вашей могиле. Не заставляйте их плакать, пока вы живы, пусть плачут, когда вы помрете.

В политической науке есть хорошо известный и изученный феномен: авторитарный или полуавторитарный режим, сталкиваясь с оппозицией, расширяет свою базу поддержки. То есть выступления против власти заставляют ее искать дополнительную опору помимо той, которой она пользовалась традиционно. Если бы термин не звучал так издевательски, можно было бы сказать, что в ответ на протесты авторитарная власть демократизируется. Хотя и не так, как этого обычно желают протестующие. Мы можем это увидеть и на примере посткрымских событий.

Я считаю всю крымскую и восточноукраинскую историю следствием или ответом власти на протесты 2011-2012 годов. Эти протесты были объективным общественно-политическим процессом: они стали манифестацией противоречий между социумом и системой управления. Во времена, как сейчас это принято называть, «сытых двухтысячных» под влиянием одних и тех же факторов — нефтяного благополучия и информационной открытости — общество прогрессировало, а аппарат управления не развивался, а в некоторых своих отсеках деградировал. Разнонаправленное развитие одного и другого — «производительных сил» и «производственных отношений», если говорить языком марксизма (аналогия не полная, но проясняющая картину), — привело к конфликту, который выразил себя в массовых протестах 2011-2012 годов. Это был процесс, который нельзя было «победить», замести под ковер или сделать вид, что его нет. Говорить о том, что протест был «слит» из-за того, что кто-то в какой-то момент неправильно себя повел, занимательно, но малоосмысленно. Понятно, что участники протестов ожидали какой-то другой реакции и других результатов для себя, но из этого не следует, что протест был «подавлен» в смысле «уничтожен». Он имел и продолжает иметь последствия.

К чему привела реакция политического режима? Для граждан и элит были предоставлены новые возможности участия в общественно-политическом процессе. При этом приобретения граждан находились почти исключительно в поле символического, тогда как элиты получили сущностные преимущества.

Как это выглядело? Власти сделали гражданам подарок — Крым. У этого события было множество внешнеполитических, внутриполитических и экономических последствий, но оно должно было вызвать — и вызвало — достаточно долгоиграющую позитивную реакцию. Присоединение Крыма было спецоперацией, которая не требовала никакого гражданского участия ни внутри России, ни в самом Крыму, тем не менее, поскольку людям все это очень понравилось, у них возникло ощущение, что они сделали что-то хорошее вместе с властью. После крымских событий вплоть до второй половины 2016 года число людей, положительно отвечающих на вопрос «Можете ли вы влиять на происходящее в стране?» постоянно росло. «Посткрымская эйфория» в строгом смысле длилась недолго — ее стерли уже осенью 2014 года первые волны экономического кризиса, — однако осознание Крыма как приобретения (а не потери или убытка) продолжается до сих пор.

После крымских событий мнение и интересы армии стали учитываться при принятии политических решений

Внутри элит определенные группы интересов получили от крымской и посткрымской политики бонусы, а значит, стали еще влиятельнее и богаче. Речь в первую очередь идет об армии и ВПК, а также об агрохолдингах и ретейле. Первые заинтересованы в агрессивной внешней политике, высоких расходах на оборону и ВПК и в собственном представительстве во власти. По сложившейся позднесоветской и постсоветской политической традиции армия и флот не являлись политическим актором. Одним из, казалось бы, очевидных, но очень редко замечаемых парадоксов нашей политической системы, начиная с послевоенного периода, является тот факт, что спецслужбы — это политический актор, а армия — нет. По итогам крымской истории и того, что последовало за ней (Восточная Украина и Сирия), армия тоже стала политическим актором, вернула, или, правильнее сказать, приобрела политическую субъектность.

Выражается это в том, что мнение и интересы армии теперь учитываются при принятии политических решений. Даже с точки зрения внешних проявлений и медиакартинки, во всех рейтингах явного и тайного влияния министр обороны будет назван в числе пяти самых значимых людей в стране. С его предшественниками такого не было. По всем циркулирующим легендам он участвовал в принятии ключевого решения по Крыму и являлся одним из основных decision-makers в дальнейшем. Кроме того, армия, которая и раньше получала много денег, теперь получает еще больше. Настолько много, что в 2016 году расходы даже пришлось немного урезать — они стали неподъемными для экономики. ВПК и армия — не единая группа интересов, но и ВПК выиграл от крымской кампании и того, что за ней последовало — прежде всего, в финансировании.

Крупные агропроизводители — сельскохозяйственные холдинги юга России, которым принадлежат целые регионы и которые в правительстве представлены министром сельского хозяйства — бывшим губернатором одного из таких регионов, — получили в качестве бонуса продуктовое эмбарго. По этой причине никакой отмены продуктовых контрсанкций в ближайшее время ожидать не следует: они слишком выгодны. К агрохолдингам присоединяется группа ретейла — сетевой торговли. Для них выигрыш чуть менее очевиден, потому что средний чек покупателей сетей уменьшился: крымская история и ее последствия отрицательно повлияли на доходы населения. Тем не менее для них сотрудничество с агропроизводителями и монополизм на внутреннем рынке, избавление от западных конкурентов — тоже фактор выгоды.

Можно выделить еще одну, менее очевидную (хотя постоянно находящуюся на виду) группу интересов — медиабюрократию, то, что называется «машиной пропаганды». Эти люди тоже получили больше, чем имели: внимание, политический статус и деньги.

Люди выбрали стратегию пассивной адаптации к новым экономическим условиям, но одновременно с этим произошло усиление политической апатии и абсентеизма

Говоря о последствиях 2014 года, важно отметить не только то, что произошло, но и то, чего не случилось. Система не перешла на военное положение, не вступила в прямой конфликт со всем миром. Ее испугала изоляция, и она стала спасаться от нее всеми методами, в том числе хаотично вмешиваясь в любые мировые процессы без какой-либо внятной цели, кроме одной — избежать изоляции. Но что еще важнее — когда за крымской историей началась история на востоке Украины, большинство граждан не приняли в ней участие. Учитывая масштабы нашей страны, поехать в Донбасс могло бы гораздо больше людей, если бы эта история действительно отвечала на внутренние потребности общества. Но этого не произошло. Участие в этих событиях российских военных спецслужб говорит о том, что добровольцев недостаточно (а сама необходимость привлечения добровольцев — о том, что было недостаточно поддержки местного населения).

Все увлекательные приключения России 2014 года не вызвали внутри страны мощной националистической волны. Можно было ожидать, что в ответ на украинские события и Крым в России приобретут популярность националистические идеи. Если бы это было так, мы бы увидели новые националистические силы: новые партии и новых лидеров. Это была бы волна такой энергии, с которой власть ничего не могла бы сделать. Но она успела вовремя кооптировать одних и репрессировать других. То, что это удалось сделать с такой легкостью, говорит еще и о том, что сущностная поддержка националистических сил была невелика.

Проще говоря, на них не было спроса. Уже по результатам региональных выборов осенью 2014 года можно сделать вывод, что те партии, которые усиленно использовали как крымскую, так и националистическую, ультрапатриотическую повестку («Родина», «Патриоты России», «Коммунисты России»), не получили за это никаких электоральных преимуществ. Парламентские выборы 2016 года этот вывод только подтвердили. Сейчас понимать это особенно важно, потому что за прошедшее с 2014 года время экономическое положение граждан ухудшилось (мы видим по динамике доходов). Люди выбрали стратегию пассивной адаптации. Но одновременно с этой пассивной адаптацией к новым экономическим условиям произошло усиление политической апатии и абсентеизма. И это станет главной проблемой выборов 2018 года: над ее решением сейчас преимущественно думает наш политический менеджмент.

Субтитры

С нами сегодня екатерина шульман политолог доцент института общественных наук российской академии народного хозяйства и государственной службы здрасте екатерина михайловна добрый день давайте все-таки давай поговорим о местном самоуправлении то что тема которая перетекает у нас и задавай эфир умных парней в другой эфир на протяжении всей недели ровно потому что это нас.

Беспокоит у нас чешется местное самоуправление это эта тема тем они в ближайшие годы мы будем говорить все больше и больше ну почему вы так уверенно потому что это базовый этаж пирамиды власти это тот ее уровень который ближе всего к людям и он касается той повестке которую людей на самом деле интересует и по мере того как люди начинают.

Осознавать связь между качеством своей жизни и решениями важных органов их взор естественно обращается на тех людей которые эти решения принимают поскольку наш с вами политическая система устроена таким образом что чем выше этаж тем меньше связь и тем парадоксальным образом по крайней мере пока выше уровень абстрактного доверия то есть претензии не обращаются там условно.

Говоря к президенту он является символом всего хорошего такой неизменный полярной звездой надежда мой компас земной вот это все а поэтому к нему пока пока по состоянию сейчас а претензий не адресуются но кому-то они должны адресоваться поэтому они адресую систем до кого можно дотянуться уровень местного самоуправления это туда чего дотянулся в принципе можно это первая.

Причина вторая причина состоит в том что при всем контроля за процессом выборов все-таки чем ниже этаж тем больше возможностей как-то влезть в эту систему и все-таки попытаться с ней хотя бы в какие-то выборные игры сыграть либо путем протестов добиться ухода какого-то человека либо избраться там хотя бы муниципальным депутатом мэром избраться все трудней и.

Трудней потому что эти выборы просто отменяются в общем как-то влезть внутрь системы и пытаться влиять на нее оттуда изнутри уровень самоорганизации возрастает люди в принципе учатся действовать вместе а поэтому они эти свои новые навыки практикуют конечно на тех кто ближе не всегда это увязывается непосредственно с местным самоуправлением я имею ввиду.

Решение проблем на местах потому что люди узнают прекрасный механизм как это работает нужно писать напрямую путину а после этого местное начальство зашевелится если выйти по протестовать не обязательно кого-то выбирать чудо можно выйти на улицу перекрыть доступ и к свалке x и после этого можно добиться отстранения главы волоколамского района правильно протесты с целью отстранения.

Кого бы то ни было это тоже вариант совместных действий что касается писание президенту после которой все шевелиться это не совсем так работает письма президенту сами по себе никого шевелиться совершенно не заставляют у нас по нашему бюрократическим правилам любая жалоба пересылается тому мы кого жалуются вот собственно и все поэтому одно письмо ничем вам не поможет.

Люди уже более-менее догадались что необходимо комплексное действие в который входят и выхода на улицу без этого никуда и письма не только президент удвоив правоохранительные органы в чем перекрестное жаловаться надо всем на всех и какая то там какая то медиакомпания привлечения публичности и компания в социальных сетях вот при наличии этих.

Инструментарий вместе то есть организации какой-то правовой составляющей в которую входит и жалобы и возможно обращение в суды и публичности есть шанс привлечь внимание к своей проблеме и что-то там решить лет пятнадцать назад помню когда внедряли 133 федеральный закон закон о местном самоуправлении было много пересудов рассуждения то как хорошо бы было решать.

Проблемы на местах и тогда этот закон по сути спускали сверху и говорили товарищи давайте сама управлять не зашло почему сейчас может за мечом подобного не говорили законы все спускают сверху так устроена наша система законотворческого процесса закон 133 меняется непрерывно это постоянно идущие реформы идет она исключительно в одном направлении объем.

Ресурсов у местного самоуправления и объем полномочий у него же и плюс к этому еще для так сказать асфальтирование ситуации это максимальный отмены прямых выборов на местном уровне так что никому сама управляться близко не позволяют вот я думаю точнее тушу без я думаю хотелось бы надеяться что сейчас вот эта вот волна протестов по самым разным причинам.

Самых разных регионов наведет архитекторов нашей политической системы на мысли что хорошо бы как то хотя бы сбросить себя ответственность распределив ее на более низкий уровень то есть отдать какую-то часть полномочий вниз всем чтобы вот вы отвечаете перед людьми чтобы можно было хотя бы сказать это не наш сфера ответственности не нас.

Федералов это вот их муниципалов вот пусть они решают сейчас занимал чего могут решать потому что у них нет денег а и б у них нет полномочий но это же делиться властью а вот тут имеется с целой харибда до некая понимаете ли catch в институт такая вот ловушка 22 конечно хочется сбросить ответственность но не хочется отдавать полномочий это вечная проблема.

Для государственной власти вот с одной стороны отвечать не хочется зубы страна распоряжаться хочется как быть не знаю по этим более если говорить о федеральной власти то у них то проблем так как будто не прибавится на прибавится проблем на местах поэтому есть ощущение что с этим прежде всего не очень согласятся на уровне регионов я между губернатор и они не будут в.

Восторге от того что муниципалитет муниципалитеты будут выделять дополнительные полномочия у них возникает политической роли в восторге от этого на они не могут быть в восторге от нынешней ситуации у вас был губернатор воробьев до было на некоторое время назад значит за любые не порядке у себя на территории отвечает все равно он и он.

Это понимает он так сказать предстоятель для за область перед федеральным центром его задача обеспечить тут некоторый уровень благолепие если благолепия не возникает или раз возникнув нарушается то виноват будет он и у него возникают проблемы с его перед назначением который у нас их и мистически называется выборами поэтому он заинтересован в том с одной стороны чтобы там как то все.

Было тихо и хорошо да а с другой стороны он был бы заинтересован в том чтобы сказать это вот муниципалитет и вот они у нас допустили и значит не решили проблему или по-крайней мере он был бы заинтересован в том чтобы были какие-то люди которые могли бы выйти и поговорить с протестующими место него по нож когда он туда выходит на это как то не очень хорошо всеми.

Воспринимается снежки и случаются на данную снежки это еще до снежки это еще все лирика вообще хочу обратить внимание дорогих слушателей на то до какой степени цивилизованны и мирно ведут себя протестующие у нас от как-то не особенно ценят у нас любое массовые собрания граждан начинает называть страшным майданом и прологом к массовым беспорядкам при этом люди ведут себя.

Идеально если хочу напомнить что сбор и мирно и без оружия это одна из базовых форм вообще говоря политической жизни граждан не абсолютно ничего я чрезвычайно ужасного и расшатываю щего вертикаль в этом нет это нормально они не жгут здания администрации они не кидаются на омон на ней росгвардию которую против них выводят они не бьют друг друга на почве.

Там несогласие в в целях они ведут себя вообще в высшей степени законопослушны а их называют какими-то вот там ужасными какими там говорил бывший губернатор тулеев бузотеры не значит цените цивилизованное поведение нашего народа она может измениться ставлю себя на место губернатора московской области воробьёва сейчас он сразу чувствуете дискомфорт и есть.

Определенный дискомфорт но я сейчас могу приехать в волоколамске и района московской области поговорить с главой района сказать что же ты человек не смог не справился будь любезен пиши заявление по собственному желанию она если не будет такой возможности она если на месте а там главы района x придет какой-нибудь глава района.

Ройзман из не уже вот такого разговора не получится вот такого разговора не получился с мэром волоколамская напомню что волоколамский ситуации это интересное там есть прямые выборы мэра а я напомню что из центров субъектов федерации из региональных столиц у нас осталось семь городов которые выбирают мэра напрямую вот сейчас отвалился екатеринбург шестнадцатом году их было.

Девять осталось осталось семь это как действительно в истинности королева произвести здесь негритят до их и становится все меньше и меньше а в городах которые не являются столицами регионов более менее где-то еще сохраняется вот волоколамск один из таких городов там сняли главу района то есть того кого могли снять вообще напомню.

То в московской области под руководством губернатора воробьёва проходила и у и продолжает проходить своя довольно масштабная муниципальная реформа в которой заменяются а вот эти вот единицы местного самоуправления на районы подчиняющиеся областному правительству это поездка губернатора это стоит о чем он в общем пришел на свою должности то что он продолжает осуществлять и в общем.

Уже практически внедрил связь между уничтожением местного самоуправления и появление вот эти вот самых массовых точечных протестов почему-то не входит в голову еще раз посмотрю архитекторов нашей политической системы а зря наличие муниципального уровня ответственного перед людьми и избираемого имя демпфирует такого рода недовольство то что все.

Замыкается исключительно на центральной власти и на уровне региона на областное правительство делает это недовольство легко проводимым и она бежит вверх по вертикали с размещать на скоростью откуда собственно говоря снежки так вот волоколамске есть хотя бы избранный мэр который с людьми ходят вместе на эти митинги и которому как мы знаем из его.

Собственных рассказов предлагали перейти на региональный уровень предлагали им это самое место для главы волоколамского района он отказался потому что вот это типа того что мой город на избирателей от них уйти не могу я так вот кажется что этот человек это вроде как как как то он противостоит значит губернатор на самом деле на самом деле он его спасает потому что он находясь вместе со своими.

Избирателями является одним из факторов того что этот весь процесс приобретает те еще раз повторю в высшей степени мирной цель удобной формы которую он пока приобретает а интересно если почитать свидетельство тех корреспондентов которые находятся на месте и в общем я рассказываю что там происходит интересно стремление людей соблюдать закон во всем вплоть до.

Мелочей им там запретили там мангалы ставить чтобы греться они перестали ставить мангал и потом человек живущий по соседству поставил монгол себя во дворе и там пускает всей греции поит чаем то есть там у нас демонстрируется вообще еще раз повторю самые лучшие качества какие тут могут быть у людей самоорганизация взаимопомощь и закону послушность но она у нас ситуативно я.

Полагал обязательно скорее она то скорее системное она может ситуативно пропадать если люди доводить до не хорошего состояния но вообще говоря будут предоставлены сами себе они склонны соблюдать закон а екатерина михайловна здесь есть тонкости волоколамска это пример проблемы и когда эта проблема уже выводят людей на улице задаются конкретные требования люди по.

Крайней мере полагают что они знают чего хотят но и если копнуть поглубже и далеко от дома не отходить то пример мой лично это у меня уже месяц не могут выбрать старшего по подъезду ровно потому что это никому в подъезде не нужно вроде тоже та же самая система самоорганизации и самоуправления но нет тех проблем с которыми сейчас нужно сражаться поэтому никому не нужен.

Дополнительный геморрой все поэтому я и говорю о ситуативности вот этого запроса на этом говорит о ситуативный закон послушности это говорит о ситуативности да потому что сама организация не возникает просто когда у него нет цели люди хотят самоорганизован средний пойми чего они не пионеры это понятно когда у вас будет проблема в.

Доме которые надо будет решить поверьте мне отлично появятся и кандидаты в кого там выбираете старше подъеду да хотя бы вот с этим разобрать понятно вообще поразительно что люди протестуют у нас возвращаясь к этой самой мусорной тематике протестуют ради того чтобы им дали право сортировать мусор и конечно поразительно вот вот вот это вот наверное не знаю где вещью в мире есть.

Такие удивительные протесты требуем разрешить вам переходить дорогу на зеленый свет нет говорит власть не поставлю вам светофор не будет вам как переходили так и будете переходить пусть вас машина забивает ну что вот это вот такое всем уже более менее понятно что следующий шаг это необходимость для сортировки мусора и его соответственно переработки всем понятно что это будет.

Это надо уже вводить более того под этот информационный фон который всех запугал и не без причины люди пойдут на это охотно они по крайне мере в крупных городах москве уж точно люди в общем к этому готовы они меньше готовы наверно к тому что будет повышена тарифы за вывоз мусора а это тоже будет они они думают что готовы потому что это пока не имеет.

Прямого отношения к ним но если вы знаете это не такая большая проблема пристегиваться научились пешеходов на переходах пропускать научились очень много чему научили за прошедшие 10 15 лет систем что люди стали менее голодные и соответственно чуть более склонные думать о несъедобным как это уехала и мусор сортировать тоже отлично научиться не надо сразу требовать чтобы.

Это был 6 разных пакетов в европе но хотя бы отделять бумагу стекло металл это абсолютно нормально и самое главное это вот эти вот так сказать и бытовые отходы пищевой мусор то что собственно и дает в эту страшную а ситуацию на свалках чё гниет откуда этот газ возникает он же не от пластика возникает да он возникает от на этих самых пищевых и пища отходов как это.

Называлось при советской власти и ну в культурных местах и вообще измельчают сначала на эти обидно думать что это же все ресурс это все та же что люди деньги платят других местах затуши вам пластик привозили которые не перерабатывают уж не говоря бумаге и картоне который весь подлежит вторичной переработке уж не говоря металле который стоит денег то.

Есть в одном месте за это платят другом месте вот как сейчас в московскую область судорожно пытается силами губернатор найти хоть какое-то место чтобы куда вы позволили свозить свой мусор и все отказываются от дороги это кстати будет интересный фактор регионального сепаратизма он типа не видите к нам ваш московский мусор манат не надо актерские уже сказал мирские.

Сказали вроде как сейчас ярославские значит принимаются ну вот как только люди об этом узнают я славлю то они тоже начнут матрасы выходить это в общем интересно москвы так не любят а тут такая наглядная вещь да то есть вроде как москва она источник денег за это мы и готовы потерпеть а когда она еще источник мусора то тут там уж вообще не готова ее любить но.

Если мусор придет с деньгами возможно местные власти диод конечно тут конечно придется да придется за это все платить но если платить только властям то люди увидят исключительно прибавку количество мусора и соответственно скажу что вот у нас будет как волоколамский мне твой не позволь вернемся к местному самоуправлению мнение екатерина михайловна шульман.

Запрос на местное самоуправление на местах есть есть он не формулируется в таком виде и люди не обязан его формулировать так и его формулирует политологи но на что есть запрос совершенно точно есть запрос на учет своих интересов при принятии решения это опять же язык политологии но я вам скажу базовый основной политический запрос это запрос на политическое участие.

А политическое участие нужно для того чтобы твои интересы принимали в расчет при принятии решений люди хотят чтобы когда принимается решение об открытии свалки а закрытие свалки о повышении тарифов о чем угодно их спрашивали это и есть политическое участие поэтому все политологи так грустно смеются когда их спрашивает а это политический протест или ты знаете.

Ли такой социально-экономический процесс или когда пишут да какой же это протеста это писание челобитных то есть нету тут настоящего про тесного такого вот знаете вот жара в душе никто не хочет опять же поджигать за ней администрации товарищи это абсолютно бессмысленное разделение не бывает ни политического протеста смысл протеста запрос на участие запрос.

На участие это запрос политический дальше тогда возникает следующий вопрос получается что вся проблема только в архитекторах политической системы которые пока не дали отмашку или что они должны дать для того чтобы эта система вообще заработала кто мешает но теоретически в москве муниципальные депутаты выбраны довольно много оппозиционных они там развернули.

Активную деятельность и никто не запрещает это делать в другими довольно трудно развернуть активную деятельность потому что у них нет полномочий в москве тоже уничтоженная система местного самоуправления смотрите когда мы говорим о том что все дело в архитектор которые не понимают как на действует изменившихся обстоятельствах определенной степени это правда они не.

Поймут пока не будет достаточно активного общественного давления не когда архитекторы ничего или точнее говоря никогда никакие политические элиты не хотят ничего менять они по определению поскольку они или ты они занимают верхнее положение в социальной пирамиды они занимают.

Максимально выгодную общественную позицию они не могут хотеть никаких изменений не когда это не в их природе поэтому когда мы говорим о нет запросу или знаете ли на реформы никогда у них не будет запрос на реформ пока припекать не начнет говоря не научным языком когда меняется социально политическая конъюнктура когда с одной стороны.

Меняются внешние условия в основном экономически с другой стороны начинается давление снизу вот тогда и политические элиты властвующие группы группы интересов начинает осознавать что чего-то надо менять еще одним фактором изменений является довольно часто конкуренция между элитными группами поэтому собственно мы все смотрим с небес удовольствие когда.

Они начинают энергично кушать друг друга конкуренция между собой является некоторым драйвером перемен одним из скажем так возможных факторов возможных этих самых дайверов с одной стороны конкуренция заставляет думать как не обезопасить себя от того кто хочет откусить мне голову с другой стороны она заставляет все будущие со стороны обращаться к общественному.

Мнению как к инструменту то есть говорить о люди то вот хотят того то или а люди меня поддерживают таким образом им кажется что они используют это самое общественное мнение или даже формирует его потому что у них есть иллюзия что небольшие властелин информационного пространства на самом деле ничего подобного в реальности они связывают себя обязательствами перед тем же самым.

Общественном мнении это еще раз сорри не единственный фактор не главный но он тоже есть его надо учитывать как путин одно из ключевых направлений работы развитие в россии конкуренции подчеркнул путин без решения задач в этой сфере ничего мы с вами не сделаем от сегодняшние это по какому поводу о чем мы там госсовет сегодня проходила и он поднял вопрос о том что нам.

Необходима конкуренция во всех сферах ну хорошо я помню я помню было друга его высказывание по поводу конкуренция как будто он говорил о том почему наши вузы в международных рейтингах занимает низкие позиции потом сказал что это же один из инструментов конкуренции поэтому они своим то типа рейтинге по лучше рисует она занижает а поэтому конкуренция может.

Пониматься своеобразно но на самом деле вообще-то говоря да и тут я как опять же глядя на на на все явления бытия из своего на профессиональные интересы скажу то политическая конкуренция тут даже важнее экономическое потому что она ее запускает все равно власти на местах пока есть выбор есть альтернатива вместо расширение полномочий можно задобрить можно задавить любую пока протест.

Договориться или запугать смотрите как это делается употребляется обычно смешанная тактика это довольно отработанные стратегии действий вообще-то говоря у нас своим протестов многое происходит они часто они не попадают в поле зрения медиа потому что медиа ждет то ли того что происходит только в москве то ли каких-то вот того что им кажется политическим то есть.

Протестов с кадровыми требова ними а поэтому на треть протесты тематически она не очень смотрит но те кто их изучаю то есть и центры исследователи которые этим занимаются и ведут эту статистику они знают что их на самом деле много самый массовый тип протеста в россии от протеста трудовые против нарушения трудовых прав их число растет довольно активно с.

Середины 14-го года самый протестный год у нас был 16-17 чуть поменьше потому что немножко снижения доходов замедлилось экономической ситуацией чуть-чуть стало получше из чем-то накормили бюджетников то есть были какие-то не а тут социальные раздачи тем не менее что делает то что делает власть когда происходит такой точный против сперс повторюсь довольно.

Отработаны и пока до поры до времени достаточно эффективная технология а по требования протестующих почти всегда идет удовлетворение то есть на самом деле то что люди требуют они получают этот факт очень сильно замалчивается что другим что называется не не стала соблазнительно но вообще-то говоря это известно то есть эти люди хотят чтобы их там не увольняли не закрывали ненене.

Снижали зарплату то это в принципе происходит а дальше а жертвуют каким-то местным чиновникам приносится ритуальное жертва и происходит точечные репрессии против активистов вот на примере волоколамска мы видим идеальный кий он стал очень публичным потому что это близко к москве б один из многих случаев и c сразу после выборов когда вот это вот.

Подмораживать ее подмораживать и ситуации перед выборами закончилось вот только выбора прошли вдруг как будто все выдохнули и тут все попова а так вот поэтому на этом примере мы можем видеть очень наглядно как это происходит но так происходит в принципе везде значит нему не удовлетворяются прием мусор прекратился говорится о том что будет там рекультивация до газация.

Приедут какие-то голландцы там поставят факел про роет скважину мне трудно сейчас судить как неспециалисту насколько это эффективная мера насколько это просто разговором о тем не менее начнут даются такие обещания и действительно перестали заезжать туда для грузовики с муссом то есть это факт идет удовлетворении требования ритуальная жертва принесена.

Кого района снят трое из наиболее активных задержанная за 14к получает 115 получает другой в в его бизнес там приходит а то есть все все как положено то есть еще раз удовлетворении требования ритуальная жертва точечный лепечешь состоялась все состоялось пока еще раз повторю это смешанная тактика в которой больше.

Уступок чем репрессии но репрессии тоже присутствует она в общем ситуативная работает когда она перестает работать когда в двух случаях когда а у систем заканчиваются ресурсы то есть удовлетворяет требование не чем проводить репрессии тоже ничем и когда количество этих случаев увеличивается настолько что она переходит в качество система не успевает всех удовлетворять в.

Московской области похоже на второй сценарий но пока он не наступит свалок много протестов тоже достаточно кроме волоколамска есть колонна в частности и поэтому там не только колонны есть прекрасная карта мусорных протестов там вот просто несколько я я думаю что дюжина а вот этих вот почек в которых протеста то ли на эти особенности происходит посмотрим внимательно пока.

Наблюдаем за процесс за процессами с нами екатерина шульман политолог доцент института общественных наук российской академии народного хозяйства и государственной службы мы сейчас расстанемся на время информационного выпуска после этого вернемся продолжим но и ближе к концу программы будем принимать телефонные звонки просто помню как можно с нами взаимодействовать пока.

Пишите плюс 7 925 8888 94 8 или через telegram говорит маска бот новость 15:35 в эфире программа умные парни у микрофона владимир карпов с нами сегодня екатерина шульман политолог доцент института общественных наук российской академии народного хозяйства и государственной службы говорим о местном самоуправлении и в продолжение темы можно поговорить о политической реформе.

Которая то ли состоится таллинне состоится сейчас выскажу свое суждение очень осторожно после выборов президента российской федерации ваши коллеги эксперты неоднократно делали заявления которые были очень похожи на вербальные интервенции о том что скорее все а не будет как такового транзита власть в российской федерации а будет реформа политических институтов российской.

Федерации для того чтобы президентский институт он слегка подразнить и его полномочия передать или другим ветвям власти или например уйти на места дать что-то побольше в власти и на местах тоже это были их суждения сейчас эта проблема на фоне запросов по местному самоуправлению как будто бы актуализировалась но я не вижу.

Пока никаких симптомов того что вот те самые архитекторы политической системы но делали хоть что-то похожее на укрепление власти на местах на делегирование полномочий или на намерение так поступить в перспективе но смотрите есть два типа реформа которых можно говорить первое это а те изменения которые в определенной степени заявлялись перед.

Выборами в ходе выборной кампании и это те реформаторские планы которые сразу разными около властными группами так или иначе писались создавались в этом отношении я бы обратил внимание на то что писал и продолжает писать центр стратегических разработок вот так сказать кудринская группа существует целый ряд других групп которые там концентрируются вокруг будут смена.

Титова которые связаны с сергеем глазьевым помощником президента все это около властные группы разной степени около властности начни какая из них не является оппозиционный при этом не какая из них не является ни какая из них не имеет стопроцентной гарантией или даже 70 процентных что их идеи будут приняты и уж тем более что они будут воплощены жизнь это первый тип это так сказать.

Сама реформаторские планы или попытки если мы доживём до попыток второй тип изменений и это то о чём тоже говорят но никогда не со ссылкой на какую конкретную фамилию никогда не патчу конкретная ответственность это изменение которое необходимо если они необходимы для реализации транзита власти то есть передача власти если мы остаемся в.

Рамках нашей пенсионной системы а политический режим наш довольно риге дин и не любит никаких резких движений никаких изменений я напомню о том что ну вот даже этот такой неудобный а параметр занятия одним лицом поста президента не больше двух сроков подряд его не не поменяли по крайней мере пока да сейчас мы конечно можем сослаться на китайский.

Пример там там убрали из устава коммунистической партии китая эту норму можно конечно ему убрать но это будет наших условиях в общем такая вот достаточно серьёзная реформа еще раз повторю система наш серьезных реформ не любит если мы вспомним какие изменения в конституцию нас вообще проводились начиная с 93 года то мы увидим что за исключением изменение связано к.

Субъектам федерации их там приращением убыванию сокращением объединением появление внезапно новых а сущностные изменения касались у лечение срок полномочий президента и государственной думы до было четыре года четыре года стало 6 лет и пять лет соответственно так вот если мы возвращаясь к началу собственно мои мысли если мы остаемся в.

Рамках нашей консольные конструкции то это строк должен быть последним для действующего президента транзит власти это ахиллесова пята всех недемократических режимов только демократии обладают великим секретом как передать власть сохранить стабильность они одни умеют это делать если бет умели делать автократии они бы завладели миром но они пока не не.

Особенно завладевает миром у них в этом месте есть проблема особенно эта проблема персоналий ских режимов то есть режимов которой сконцентрированы и в которых власть и ресурсы концентрируется вокруг одного лица за называется для протокола я не считаю наш с вами политический режим до такой степени персонале скин как обычно полагают я считаю что власть у нас с вами.

Принадлежит распространенной бюрократии бюрократическому классу вооруженной бюрократии гражданской бюрократии силовой бюрократии экономической я не думаю что у нас на до такой степени вся система заточена под одного-единственного человека мне кажется что это скорее картина которая создает силами пропаганды и на самом-то деле транзит власти возможен может быть.

Великая тайна заключается в том что он вообще легче пройдет чем все полагают тем не менее к чему коллективный разум система через некоторое время придет какой без какой реальностью им придется столкнуться реальность состоит в том что тот объем власти который был у президента которому полагал в периоды молодости и силы нашего политического режима.

А это уже не сегодня это уже был некоторое время назад свою высшую точку он прошел он непередаваем в полном объеме его распределение в некоторой степени уже происходит это то что мы последние годы наблюдать не концентрация распределения между силовыми группами в основном это не очень хороший сценарий с точки зрения благо россии с точки зрения пользы для общества что власть до такой.

Степени находится у силовых акторов тем не менее мы сейчас не будем раздавать не команд мы просто пытаемся придать реальность как какова она есть так вот это распределение уже происходит в ползучим виде если мы будем говорить о том что власть нужно передать от лица x кому-то другому то выяснится довольно скоро с такого лица y не может существовать элитной группы не.

Договорятся о каком-то одном лице они будут бояться передачи такого объема власти одному человеку соответственно необходимо какое-то распределение она может приобретать самой разной формы ну вот есть все эти мутные разговоры о том что давайте мы за счет госсовет накачаем дополнительными полномочиями у нас будет представителю совета национальный лидер и присвоим ему имя начинаем сяопин и он.

Будет править вечно а дальше будет президент чуть чуть менее полновластный чуть более технически я еще будет премьер и личинки что другое придумаем или премьера вообще уничтожим у нас будет как в америке да нет правительства есть расширенная президентская администрация это подается под лозунгом давайте если туз анализируем то что у нас и так происходит если у нас такая.

Система придворное давайте мы этот двор и в качестве правительства для собственно говоря и установлен разные есть идеи это можно называть публичном пространстве чуть ли не там созданием какой-то монархической формы сумм будет ровно то же самое это все равно будет распределение власти между различными группами и различными органами.

Если это будет какой то коллективный орган типа великого совета не знаю как его назвать госсовет у нас сейчас существует но не особо функционирует и совет безопасности очень интересное сашин не публичную структур в которой как считается принятие ряда ключевых решений в особенности внешнеполитических происходит может быть он станет новым центром власти и это все пока довольно.

Туманно и разговору потому что имейте система живет очень коротким горизонтом планирования и и задачу ночь простоять день продержаться и вообще-то говоря ничего не менять то о чем мы с вами говорили до перерыва на рекаму да это некая абсолютная реальность властвующие группы не хотят никаких изменений они хотят всю сохранить как есть когда они сталкиваются с.

Противостоянием самой жизни с тем что нельзя остановить часовую стрелку нельзя заморозить время то-то они начинают пытаться как немножко измениться тем чтобы как бы вы что-то поменять чтоб ничего не поменять поэтому эти изменения будут носить не планируемый характер не характер каких-то стратегических реформ а я думаю что характеру очень ситуативный то есть это самый властная.

Вертикаль будет а плевать как некий оседающие suhrob но она не будет осознавать что с не это происходит это не будет заявлено виде какого-то реформаторского курса с одной стороны с другой стороны я думаю что какие-то точечные реформы из того же кубинского набор мы с вами увидим по крайней мере по про декларируемыми в ближайшие два года но тогда это выглядит как процесс.

Абсолютно неуправляемое а те же самые предложения звучат исключительно как благопожелания все не более того такая картина вырисовывается снова уже это будет выглядеть как неуправляемые процессы внутри будет казаться что вот мы удерживаем ситуацию мы и вот прям почти совсем уже удержали от уж там какие-то смутные особи пока нем-то мы договорились не обращать на.

Это внимание я бы скорее представлял себе такую картину и конечно одновременно вот это внутри конкуренция будет усиливаться мы это уже видим потом всяким последним обском и посадка мы увидим этого добра еще много силовики будут есть друг друга не не связанного с силовыми акторами бизнеса в общем не существует поэтому не надо представлять картину так что злые силовики значит.

Селе травоядных коммерсантов нет никаких травоядных коммерсантов это clientele a1 силовой группы или клин тела другой силовой группы и вариантов в общем уже по итогам последних шести восьми лет на самом деле не осталось до они будут продолжать поедать друг друга но эта борьба за власть тогда зарплата не оставляем это борьба за ресурсы и.

Ресурсы власти они близки товой она в любом случае тогда оставляет за скобками все те благопожелания которые были высказаны в том числе и центром стратегических разработок алексей кудрин а вот и все но что получается у нас мы с вами ведущая часть нашей беседы говорили о том что эта внутренняя конкуренция может быть одним из драйвером в драйверов изменения она является им.

Примите в борьбе каждая из групп в общем ослабевает еще раз повторю они все пытаются апеллировать к общественному мнению какому-то общественному благу хотя бы на уролог вариаций хотя бы на уровне декларации вот эти вот провозглашенные лозунги ускорение перестройки но перестройка у нас не провозгласили ускорение провозгласили да они должны быть выражены в каких-то.

Властных решениях то что было сказано в послании президента я имею ввиду первую часть так сказать гражданскую не вторую часть военную они должны быть реализованы это законы бюрократического бытия до из послания делаются поручение из поручения делаются постановление правительства проекты законов которые носятся государственному что-то из этого должно быть каким-то.

Образом осуществлена и тут это самая кудринская повестка еще раз повторю очень точечно обнаружит себя виде до каких административных решений времени остается 15 минут обычно это время как распределяем вопросам вопросам нашей аудитории предлагаю сейчас к слушателям и обратиться на помню как можно с нами взаимодействовать телефон.

Прямого эфира 737 394 8s плюс 7 925 8888 94 8 работает наш telegram говорит маска бот поехали слушаем сейчас вас внимательно да здравствуйте добрый день добрый вы сели пожалуйста екатерина вы знаете но говорить вы вещи и все такое но мне кажется что жизнь немножко чуть-чуть как бы сказать примитивнее чем вы говорила гаити вот если вы вы наблюдали.

За наши дома это видели в выступлении воробьёва на заседании госдумы где он очень здорово навешал так грубо выражаясь юга новую жириновскому и вот то что произошло в волоколамске это во многом не кажется мне с воробьем со стороны особенно последнего товарища дуганов и жириновский организовали с начала войны на свалке а потом выгнали.

Людей представать но вы знаете это очень знакомый подход это конспирологическое мышление им страдают почти повально все властные акторы и к сожалению некоторые граждане тоже потому что это тот диску с которой не слышат в больших количествах из средств массовой информации поэтому они все повторяют нельзя организовать социальный процесс.

Это во первых во вторых если хотите разозлить специалиста в любой области знания скажите ему словно просто ничего не просто говорил борхес единственное что мы знаем о вселенной это то что оно бесконечно сложна социальные процессы чрезвычайно сложная российская федерация сверхсложное образование то что у нас происходит абсолютно не просто поэтому когда вы начинаете разговаривать с.

Экспертами вам кажется что он говорит а не то чтобы невнятно но как-то очень уж общо потому что к шарлатаны дают точные прогнозы только шарлатаны дают определенные оценки если ты хочешь оставаться в рамках базовой научной добросовестности ты вынужден будешь говорить достаточно скажем так на но на довольно высоком уровне обобщения поэтому нет зубов и.

Жириновский ними игроков а отношения к тому же происходит в волоколамске более того смотрите как откуда возникает такого рода вещи действительно есть губернатор воробьев есть любой губернатор на свете у него есть враги есть другие властные actor которые желают ему зла хотят сместить его должности подвести опыт уголовное дело и так далее они используют любой.

Повод для усиления своих позиций и для продвижения этой самой своей цели из этого не следует что они есть причина выступления причин выступлений довольства людей но как только возникает такого рода очаг недовольство тут же туда естественно приска кивают разные заинтересованные люди в большинстве случаев они никак не влияют на происходящее и ничего не делаю.

Так пишет вы на нем exceleram каналах но поскольку внутри власти люди видят только друг друга только других важных факторов они граждан то и они принимают следствие за причину им кажется анонимно телеграм-канал это и есть вот вот собственно все что происходит в волоколамске потому что они обращают внимание друг на друга это естественно на печально юра.

Каменков пишет случае с ройзмана показал что если нет фундамента из сильного независимого местного самоуправления то можно повернуть политический процесс вспять по щелчку и мнение профессионалов мнений граждан никого особенно не интересует ситуация петербурге очень печальная это было плохое бессмысленное решение влияет не так часто говорю потому что не так часто.

Случаются какие-то однозначные события обычно любое явление в общественно физическом пространстве имеет много разных сторон так вот отмену прямых выборов это просто плохо и все тут никакой не к положительной стороны тут найти невозможно они отобрали полномочий у избранного мэра и могли бы же успокоиться на том что люди могут выбирать себе хотя бы такого вот.

Публичного заступника в общем откровенно говоря функция ройзмана сводил с тому что он вел прием населения я пытался людям как-то помогать опираясь не на свой властный ресурс который у него нету она свою известность и связи то есть он вел себя как публичная фигура они как администратор потому что качество регистратору не он был лишён и рычагов вот он вел прием.

Граждан и писал об этом фейсбуке и даже это рассматривается властью как угрозу ночную теперь они получили общую точку недовольство екатеринбург город очень активный с богатой политической традиции с хорошей оппозиционные организации с довольно большим количеством людей известно в том числе на федеральном уровне политических активистов так что в общем ну чё ну ну ну зря зря.

Не это сделать но пока протест выглядел довольно black даровано поэтому и вы особенно то и не заметить а значит он выглядел вполне для города екатеринбурга хорошо значит там было довольно много народу и это только начало александр пишет если пас если позволите пару вопросов по грядущем выборам мэра москвы 1 есть ли у нашей московской оппозиции шанс все-таки провести полноценные.

Праймериз выдвинуть единого кандидата или опять товарищи передерутся мной 2 будет ли следующем в следующем электоральном цикле в москве то же что и в екатеринбурге смысле отмены прямых выборов мэров но как версия что такое может случиться например ну городам так съесть федерального значения все-таки решено дать право выбирать себе мэров самим и там мэр от настоящей мэр он.

Обладает если не все это в общем серьезной полнотой власти он обладает поэтому нет пока не похожа за отдаленное будущее говорить не будем пока не похоже не видно признаков к тому что касается выборов мэра москвы это интересная история начну пока существует муниципальный фильтр то вот этот основной инструмент не допуска котором власть в общем.

Обеспечивает себе нужные результаты выборов он действует и в москве то есть можно не допустить кого угодно не исключено что в москве выбор пойдут по президентскому сценарию то есть ну так сказать им волк и семеро козлят до белоснежка и семь гномов а кандидата от власти и некоторое количество комических фигур так было сделано на президентских выборах.

Посмотрим правде в глаза и это был один из факторов совершенно не единственный и не главный главный фактор это был снос избирателей неосознанных по месту работы но когда их уже с 1-ым показали бюллетень в котором у них какого вообще и серьезных людей взрослых не была возможность проголосовать ни за кого другого кроме как за основного кандидата не исключено что в москве будет тот же.

Сценарий но москве все таки не президент россии он лидер и так сказать или чем одной из групп интересов есть другие группы интересов которые они заинтересованы в том чтобы ему устраивать такие льготные условия то есть он все-таки еще раз повторю не президент почему ему должно быть позволено вот так вот легко избираться вообще не участвую забирать на компании.

Как это сделал президент россии поэтому а будут другие группы и это не московские оппозиционеры и даже не московские граждане что грустно которые захотят чтобы ему служба медом все-таки не казалось поэтому какие-то кандидаты представляющая альтернативу не комическую альтернативу могут быть зарегистрирован тогда мы можем пытаться.

Повторить историю в 13-м году как это был на выборах мэра москвы в 13-м году протестный потенциал в москве очень высокий москва столица российской федерации тут самый грамотный самый организованы электорат самое высокое проникновение интернета социальных сетей оппозиционных сми иностранных сми то есть тут называется можно тут можно работать но и это же.

И эти же самые обстоятельства они эту работу и усложняют я не уверена пока на этапе сейчас что в движение единого оппозиционного кандидата это обязательно хорошая тактика может быть это хорошая тактика а может быть и нет тут надо думать никакие мемы и таскать трупы типа объединение оппозиции или храм наоборот разъединения позиций не должны быть.

Руководством действию пока мы не подумали о целях и о методах достижения этих целей это второй так сказать этап этого самого думания то есть исходить надо из того что кандидат от власти победит это реальность которой надо считаться если мы будем строить нереалистические планом и ни к чему хорошему не придем и второй момент с чего можно добиваться мне показалось что.

Можно добиваться выхода во второй тур или подвода ситуации когда они второго тура как это было в тринадцатом году это важная цель если она может быть достигнуто потому что мэр который выиграл с небольшим перевесом который еле еле избежал второго тура это совсем не тот кто выиграл 7 10 процентами а его ближайший преследователь получил восемь процентов.

Это разные две разные политические ситуации новые уже описывали тактику который может последовать после а именно удовлетворении требований дальше у нас что там следует принесение ритуальные жертвы ну и третий пункт это разогнать самых оттович и точечные репрессии но эта тактика которая употребляется для русских за гашения протестов тут нас немножко другая.

Ситуация вот результат выборов тоже может быть воспринято как некий протест а то что будет после дала например вы говорите о том что это будет важный сигнал для власти и если там кандидата от власти победит не с результатом 70 процентов а 50 или около это для того чтобы последовал такая реакция как вы описываете вот этот вот результат должен быть воспринят как общая угроза общая.

Угроза для властной машина еще раз повторю мэр москвы это один из политических акторов есть другие политические акторы которые добра ему не желают этот парадоксальный образом гражданское общество становится союзником высшей степени неприятных людей которые не желают ему добро но они тоже не желают добра и своему так сказать коллеги по.

Этому элитному клубок до политический процесс в не демократиях выглядит в общем так я приходится солидаризироваться в в неявной форме с теми людьми с которыми ты вообще зайдем с пол не сел бы в реальной жизни никогда не каком случае но они не хотел садиться тобой за один стул просто у вас время на тактически совпадать или 737 394 8 телефон прямого.

Эфира слушаем вас внимательно да здравствуйте добрый день и как кадра катить каноника общем как открывать новую как и предмета марки brent уфе думаю нет его надеть коневод выполнены как бы глубине души мне казалось что уже проехали с майскими указами нет смысла заказами не проехали до конца потому что они принять и беда-то в том не в том что их не выполняли.

Игнорировали их выполняли как могли и это практически переломила хребет региональным бюджетом и это загнал их в другие регионы с которыми теперь минфин пытается разными способами справляться и у нас есть мы посмотрим бюджетные планы на ближайшие три года у нас раздел который отвечает за ведь самый внутренний добьемся увеличивается и доходит в бюджете 19 и 20 годов до 20.

Процентов довольно много проблем не в том что не выполняют отношу выполняют как могут и от этого происходит впоследствии которых никто почему-то заранее не предполагал сейчас у нас какая-то новая порция раздачи всяких социальных пряников планируется поскольку ни одна избирательная кампания без этого не обходится кажется что учитывая и таскаете реальности.

Экономической конъюнктуры обошлось малой кровью то есть каким то разовыми выплатами и вот этим вот внезапным повышением зарплат труженикам бюджетного сектора которые вдруг обнаружили что они получили в два в три раза больше чем обычно но это счастье видимо продлится два-три месяца потом прекратится с точки зрения людей это грустно с точки зрения общего благополучия в общем наверное.

Хорошо что вот не случилось второй серии майских указов то есть такой масштабной раздаче слонов которых между нами говоря нету давайте еще дальше тогда один вопрос 737 394 8 слушаем тогда вас внимательно да михалыч добрый вот смотрите давайте мы еще вернемся к тонкому старые тогда литром подобно ты сломаешься глава администрации московской области.

Как вы называете губернатора я вот никогда не загубили милена только покороче михайлович рози почему он говорит что вот будем строить вчера матрас сжигают зарабатывающие заводы но раз хочется будем мы налогоплательщики говори когда говорит это представитель московской области ты имеешь ввиду что москве и московской области.

Почему мы не слышим насчет мусора главу или как вы говорите мэра г москвы мусор московский почему г москва не строит тебе мусороперерабатывающие любят 3 не заводы принимается принимается на почему мусор у одних а расходы у других потому что существует ну потому что никто не хочет нести расходы понятное дело все хотят увести свой мусор куда нибудь заплатить за вывоз и забыть об.

Этом на территории города мусоросжигательные заводы строить действительно затруднительно для этого они должны быть такие как в японии то есть и от которых вообще в атмосферу ничего не выходит а для того чтобы мусор ожидающие заводы были безопасны на них должен двигаться только отсортированный мусор это опять приводит нас к проблеме сортировки мусора точнее проблеме а к.

Неизбежности к тем же японцам которые даже бутылочки моют перед тем как бросить в мусор ровно потому хотя бы до нам хотя бы пока дойти до того чтобы бутылочку отдельно баночку отдельно отдельно бумагу и картон и отдельно мусор органический если вот до этого дойдет то у нас уже на станете не видно и счастье в нашей прекрасной российской федерации опускаясь с высот.

Необходимость страшный болото политической конъюнктуры и до существует ли не конфликта между москвой и московской областью а и это тоже один из драйверов происходящего в эпоху конфликтов мы как будто бы вступаем ровно потому что вы давно уже в ней находимся буду продолжать в ней находиться а еще это накладывается на эпоху всеобщей.

Прозрачности поэтому весь этот террариумы его обитатели видны общество очень хорошо они сами не осознают до какой степени они хорошо видны прекрасно я буду нас ожидает с нами была екатерина шульман

Больше года назад мы начали с интересом следить за публикациями и выступлениями политолога Екатерины Шульман: нас увлекла здравость ее суждений и ясность языка. Некоторые даже называют ее «коллективным психотерапевтом». Мы пригласили эксперта в редакцию, чтобы разобраться, как возникает этот эффект.

Psychologies:

Есть ощущение, что в мире происходит что-то очень важное. Глобальные перемены, которые у одних вызывают воодушевление, а у других – беспокойство.

То, что происходит в глобальной экономике, часто именуют «четвертой промышленной революцией». Что под этим подразумевается? Во-первых, распространение робототехники, автоматизация и информатизация, переход к тому, что называют «экономикой посттруда». Человеческий труд приобретает иные формы, раз промышленное производство, очевидно, переходит в крепкие руки роботов. Основной ценностью будут не материальные ресурсы, а добавленная стоимость – то, что добавляет именно человек: своим творчеством, своей мыслью.

Второе направление изменений – прозрачность. Приватность, как ее понимали раньше, от нас уходит и, видимо, уже не вернется, мы будем жить на публике. Но и государство тоже будет для нас прозрачным. Уже сейчас во всем мире открылась картина власти, в которой нет никаких сионских мудрецов и жрецов в мантиях, а есть растерянные, не очень образованные, корыстные и не вызывающие большой симпатии люди, которые действуют исходя из своих случайных побуждений.

Это одна из причин происходящих в мире политических изменений: десакрализация власти, лишение ее священного ореола тайны.

Кажется, что вокруг все больше некомпетентных людей.

Интернет-революция, и особенно доступ к интернету с мобильных устройств, привела в публичную дискуссию людей, которые раньше в ней не участвовали. От этого возникает ощущение, что везде полно неграмотных людей, которые несут чушь, и любое дурацкое мнение имеет такой же вес, как мнение обоснованное. Нам кажется, что толпа дикарей пришла на выборы и голосует за других таких же. На самом деле это и есть демократизация. Раньше в выборах участвовали те, у кого был ресурс, желание, возможности, время...

И какой-то свой интерес...

Да, способность понять, что происходит, зачем голосовать, какой кандидат или партия отвечают их интересам. Для этого требуется довольно серьезное интеллектуальное усилие. За последние годы уровень благосостояния и образования в обществах – особенно в первом мире – радикально вырос. Информационное пространство стало открыто для всех. Не только право на получение и распространение информации, но и право высказаться получили все.

Что я считаю основанием для умеренного оптимизма? Я верю в теорию снижения уровня насилия

Это революция, сравнимая с изобретением книгопечатания. Однако те процессы, которые мы воспринимаем как потрясения, на самом деле не разрушают социум. Происходит перенастройка власти, системы принятия решений. В общем, демократия работает. Привлечение новых людей, ранее не участвовавших в политике, – испытание для демократической системы. Но я вижу, что пока она его выдерживает, и, думаю, в конце концов выдержит. Будем надеяться, что жертвами этого испытания не станут системы, которые еще не являются зрелыми демократиями.

Как может выглядеть осмысленная гражданская позиция в условиях не очень зрелой демократии?

Тут никаких секретов и тайных методов нет. Информационная эпоха дает нам большой набор инструментов, помогающих объединяться по интересам. Я имею в виду гражданский интерес, а не собирание марок (хотя последнее тоже хорошо). Ваш интерес как гражданина может состоять в том, чтобы у вас не закрывали больницу по соседству, не вырубали парк, не строили башню во дворе или не сносили что-то, что вам нравится. Если вы работаете, то ваш интерес в том, чтобы ваши трудовые права были защищены. Поразительно, что у нас нет профсоюзного движения – притом что большинство населения работает по найму.

Взять и создать профсоюз непросто...

Можно хотя бы задуматься над этим. Осознать, что его появление в ваших интересах. Это и есть та связь с реальностью, к которой я призываю. Объединение по интересам – это создание той сетки, которая заменяет нам слаборазвитые и не очень хорошо действующие государственные институты.

С 2012 года у нас проводится общеевропейское исследование социального самочувствия граждан – Евробарометр. Оно изучает количество социальных связей, сильных и слабых. Сильные – это близкие отношения и взаимопомощь, а слабые – только информационный обмен, знакомства. С каждым годом люди в нашей стране говорят о все большем количестве связей, как слабых, так и сильных.

Наверное, это хорошо?

Это настолько улучшает социальное самочувствие, что даже компенсирует недовольство государственной системой. Мы видим, что не одиноки, и у нас возникает несколько неадекватная эйфория. Например, тот, у кого (по его ощущению) больше социальных связей, более склонен брать кредиты: «Если что – мне помогут». А на вопрос «Если вы потеряете работу, легко ли вы ее найдете?» он склонен отвечать: «Да в три дня!»

Эта система поддержки – в первую очередь друзья в социальных сетях?

В том числе. Но связи в виртуальном пространстве способствуют росту числа связей и в реальности. К тому же ушел советский государственный прессинг, который запрещал собираться втроем, даже чтобы почитать Ленина. Выросло благосостояние, и у нас стали надстраиваться верхние этажи «пирамиды Маслоу», а там есть и потребность в совместной деятельности, в одобрении со стороны ближнего.

Многое из того, что для нас должно делать государство, мы устраиваем себе сами благодаря связям

И опять-таки информатизация. Раньше как было? Человек уезжает из своего города учиться – и все, он туда вернется только на похороны родителей. На новом месте он с нуля создает социальные связи. Сейчас мы свои связи возим с собой. И новые контакты заводим гораздо легче благодаря новым средствам коммуникации. Это дает ощущение контроля над своей жизнью.

Такая уверенность касается только частной жизни или государства тоже?

Мы становимся менее зависимы от государства благодаря тому, что мы сами себе министерство здравоохранения и образования, полиция и погранслужба. Многое из того, что для нас должно делать государство, мы устраиваем себе сами благодаря связям. В результате парадоксальным образом возникает иллюзия, что дела идут неплохо и, значит, государство работает хорошо. Хотя мы к нему нечасто обращаемся. Допустим, не идем в поликлинику, а врача вызываем частным образом. Детей отдаем в ту школу, которую рекомендовали знакомые. Уборщиц, сиделок и помощников по дому ищем в соцсетях.

То есть мы просто живем «среди своих», не влияя на принятие решений? Лет пять назад казалось, что сетевая активность принесет реальные перемены.

Дело в том, что в политической системе движущей силой является не индивидуум, а организация. Если вы не организованы, вас не существует, у вас нет политического бытия. Нужна структура: «Общество защиты женщин от насилия», профсоюз, партия, союз озабоченных родителей. Если у вас есть структура, вы можете производить какое-то политическое действие. Иначе ваша активность носит эпизодический характер. Вышли на улицы, ушли. Потом еще что-то случилось, снова вышли.

При демократии жить выгоднее и безопаснее по сравнению с другими режимами

Чтобы обладать продленным бытием, нужно иметь организацию. Где нашему гражданскому обществу удалось добиться наибольших успехов? В сфере социальной: опека и попечительство, хосписы, обезболивание, защита прав пациентов и заключенных. Изменения в этих сферах происходили под давлением прежде всего некоммерческих организаций. Они входят в легальные структуры типа экспертных советов, пишут проекты, доказывают, объясняют, и через некоторое время при поддержке медиа происходит изменение законов, изменение практик.

Политическая наука дает вам сегодня основания для оптимизма?

Смотря что вы называете оптимизмом. Оптимизм и пессимизм – понятия оценочные. Когда мы говорим об устойчивости политической системы – это внушает оптимизм? Одни боятся переворота, а другие, может, только его и ждут. Что я считаю основанием для умеренного оптимизма? Я верю в теорию снижения уровня насилия, предложенную психологом Стивеном Пинкером. Первый фактор, который приводит к снижению насилия, – как раз централизованное государство, которое берет насилие в свои руки.

Есть и другие факторы. Торговля: живой покупатель выгоднее мертвого врага. Феминизация: все больше женщин участвуют в социальной жизни, растет внимание к женским ценностям. Глобализация: мы видим, что везде живут люди и нигде они не с песьими головами. Наконец, информационное проникновение, скорость и легкость доступа к информации. В первом мире фронтальные войны, когда две армии воюют друг с другом, уже маловероятны.

То есть самое страшное уже позади?

Во всяком случае, при демократии жить выгоднее и безопаснее по сравнению с другими режимами. Но прогресс, о котором мы говорим, не покрывает всю Землю. Могут быть «карманы» истории, черные дыры, в которые проваливаются отдельные страны. Пока жители других стран наслаждаются XXI веком, там процветают убийства чести, «традиционные» ценности, телесные наказания, болезни и нищета. Ну что сказать – не хотелось бы оказаться в их числе.